Vous êtes sur la page 1sur 82

Olimpiada de Matem aticas del Distrito Federal

Folleto de Problemas 2011


Leonardo Ignacio Martnez Sandoval

Ultima Modicacion: 5 de diciembre de 2010

Indice general

Indice General I
1. Introduccion 3
1.1. Acerca de la Olimpiada . . . . . . . . . . . . . . . . . . . . . . . . . . . . . 3
1.2. Mensaje para el participante . . . . . . . . . . . . . . . . . . . . . . . . . . 4
2. Enunciados de los Problemas 7
2.1. Primera Etapa . . . . . . . . . . . . . . . . . . . . . . . . . . . . . . . . . . 7
2.2. Segunda Etapa . . . . . . . . . . . . . . . . . . . . . . . . . . . . . . . . . . 13
2.3. Tercera Etapa . . . . . . . . . . . . . . . . . . . . . . . . . . . . . . . . . . . 18
2.4. Cuarta Etapa . . . . . . . . . . . . . . . . . . . . . . . . . . . . . . . . . . . 19
2.5. Quinta Etapa . . . . . . . . . . . . . . . . . . . . . . . . . . . . . . . . . . . 20
2.6. Examen Regional . . . . . . . . . . . . . . . . . . . . . . . . . . . . . . . . . 22
2.7. Examen de Desempate . . . . . . . . . . . . . . . . . . . . . . . . . . . . . . 23
3. Soluciones de los Problemas 25
3.1. Primera Etapa . . . . . . . . . . . . . . . . . . . . . . . . . . . . . . . . . . 25
3.2. Segunda Etapa . . . . . . . . . . . . . . . . . . . . . . . . . . . . . . . . . . 31
3.3. Tercera Etapa . . . . . . . . . . . . . . . . . . . . . . . . . . . . . . . . . . . 36
3.4. Cuarta Etapa . . . . . . . . . . . . . . . . . . . . . . . . . . . . . . . . . . . 39
3.5. Quinta Etapa . . . . . . . . . . . . . . . . . . . . . . . . . . . . . . . . . . . 43
3.6. Examen de Desempate . . . . . . . . . . . . . . . . . . . . . . . . . . . . . . 52
4. Examenes de Practica de Temas Especcos 57
i

INDICE GENERAL 1
5. Problemas de Tarea 61
6. Herramientas

Utiles 65
2

INDICE GENERAL
Captulo 1
Introducci on
1.1. Acerca de la Olimpiada
La Olimpiada Mexicana de Matematicas es un concurso que se organiza cada a no
con el n de que los estudiantes de Bachillerato y Secundaria estimulen su gusto por las
matematicas. Esto trae a su vez numerosas consecuencias. Los jovenes pueden encontrar
compa neros que compartan su gusto por esta area de la ciencia. As mismo, la Olimpiada
Mexicana de Matematicas ayuda a detectar a jovenes talentosos en esta disciplina y a
difundir que tipos de problemas enfrenta un matematico en su profesion.
Los problemas de la Olimpiada son un poco distintos a los problemas escolares,
pues usualmente en estos ultimos el estudiante requiere conocer una gran cantidad de
teora conforme va avanzando en su a no escolar. Los problemas de Olimpiada simplemente
requieren un entendimiento basico de los conceptos escolares. La principal diferencia es
que los problemas de Olimpiada para ser resueltos requieren un poco mas de instinto,
gusto por las matematicas y una peque na dosis de creatividad.
La Olimpiada Mexicana de Matematicas esta organizada en las siguientes etapas:
Etapa Estatal: Esta es la etapa en la cual se entra a la Olimpiada. En esta etapa
los estados realizan distintas pruebas para elegir a los participantes que asistiran al
concurso nacional de matematicas. Durante esta etapa los estados tambien preparan
a sus estudiantes por medio de entrenamientos a los cuales puedes asistir una vez
pasada la primer etapa. Dependiendo del estado en donde vivas, tendras que inscri-
birte con el comite organizador local. Puedes encontrar una lista de los encargados
de la Olimpiada en tu estado en http://erdos.fciencias.unam.mx/omm/. En el Dis-
trio Federal tenemos la pagina www.matematicas.unam.mx/omdf, en la cual puedes
estar al tanto de la convocatoria y las inscripciones.
Concurso Nacional: En esta etapa se invita a la delegacion de cada estado a pasar
una semana en el estado organizador. Los alumnos invitados a este concurso se les
3
4 CAP

ITULO 1. INTRODUCCI

ON
proporciona hospedaje, transporte y alimentacion. El Concurso Nacional consiste en
una prueba a dos das. Cada examen es de 3 preguntas. El resto de la semana se
dedica a la calicacion de las pruebas y a distintas actividades que el estado sede
organize. En la premiacion se entregan primeros, segundos y terceros lugares en
proporcion 1 : 2 : 3, en donde la cantidad total de personas premiadas debe ser lo
mas cercano posible a la mitad de los participantes.
Entrenamientos Nacionales: Las personas que obtuvieron Primer Lugar en el
Concurso Nacional son invitadas a formar parte de la Pre-Seleccion nacional mexi-
cana, y son candidatos a participar en los distintos concursos internacionales en los
que Mexico esa invitado, como son la Olimpiada Centroamericana de Matematicas,
la Olimpiada Iberoamericana de Matematicas, la Olimpiada de la Cuenca del Pac-
co y la Olimpiada Internacional de Matematicas. A estas personas se les entrena
intensivamente en el transcurso del a no, y deben realizar varios examenes elimina-
torios para nalmente formar parte de la delegacion mexicana en alguno de estos
concursos.
1.2. Mensaje para el participante
Bienvenido a la Olimpiada de Matematicas del Distrito Federal!
A no con a no contamos con j ovenes entusiastas interesados en la resolucion de pro-
blemas matematicos. Estos jovenes, como t u, saben que las matematicas son una ciencia
en la cual se pueden encontrar problemas interesantes, divertidos y difciles. Todos ustedes
son el principal motor de la Olimpiada, y en cada una de nuestras ediciones trabajamos
para que aprendan cosas interesantes y convivan con otros compa neros que tambien les
interesa esta bella disciplina.
Este folleto muestra el trabajo que se realizo el a no pasado en la Olimpiada del
Distrito Federal. Ayuda a que te des una idea de que tipos de problemas te puedes en-
contrar cuando participese en las distintas etapas. La primer parte del folleto enuncia los
problemas que se plantearon a los estudiantes el a no pasado.
En las primeras etapas los examenes son de opcion m ultiple, o simplemente requieren
una respuesta numerica. Te recomendamos intentar estos problemas pues son los que
inician el camino de las Olimpiadas de Matematicas. Usualmente la resolucion de esta
primer etapa requiere una buena dosis de ingenio, habilidad para encontrar patrones y
una mente paciente que no caiga en las trampas que se presentan en cada problema.
Las siguientes etapas requieren un poco mas de dedicacion. La resolucion de los
problemas no es mucho mas dicil que en las primeras etapas. Sin embargo, al ir avanzando
en la olimpiada debes aprender a explicar tus pensamientos, y a justicar de manera un
poco mas formal tus intuiciones y descubrimientos.
Puedes sentir que estos problemas son mas difciles. En realidad muchos de estos pro-
blemas simplemente necesitan algunas tecnicas que todava no has aprendido. Si intentas
1.2. MENSAJE PARA EL PARTICIPANTE 5
estos problemas y no puedes resolverlos, no te desanimes. La mayora de los concursantes
no saben que es lo que tienen que hacer, y es por eso que existen los entrenamientos, que
son clases que se imparten en la Facultad de Ciencias a los concursantes que pasan las
primeras etapas. Durante estas sesiones los entrenadores del Distrito Federal te ense naran
estas tecnicas que puedes utilizar y te apoyaran a que te familiarizes con las situaciones
en las que son utiles.
Una vez que hallas intentado un poco los problemas, puedes consultar las soluciones
al nal del folleto. Estas respuestas tambien son una fuente generosa de informacion, pues
en ellas puedes encontrar ideas que no habas visto antes. As mismo, leer estas soluciones te
guiara en el camino hacia la escritura de tus ideas para cuando llegues a etapas posteriores.
Finalmente, en la ultima parte de este folleto encontraras todava mas herramientas
para la resolucion de problemas. Estas herramientas se dividen en dos tipos. Por un lado,
la mayora de las observaciones de esta seccion son muy intuitivas, pero son muy poderosas
para resolver los problemas de las distintas etapas. Otras de las herramientas son hechos
un poco teoricos como propiedades de los exponentes y trigonometra que te serviran como
referencia cada vez que necesites recordarlas.
Esperamos que te animes a dar el primer paso presentando el primer examen. Si te
gustan las Matematicas, o estas descubriendo si te gustan, la Olimpiada de Matematicas
es de las mejores experiencias que puedes tener.

Animo y te esperamos!
Leonardo Ignacio Martnez Sandoval
6 CAP

ITULO 1. INTRODUCCI

ON
Captulo 2
Enunciados de los Problemas
En esta seccion puedes encontrar los problemas de los examenes de la edicion pasada
de la Olimpiada. Estos problemas te pueden servir de practica y para que te des una idea
de que tipo de problemas te puedes encontrar en la Olimpiada. Las primeras etapas son
problemas de opcion m uliple, en los que te puedes apoyar en la eliminacion de opciones y
en tu intuicion. Los problemas de etapas posteriores requieren un poco mas de dedicacion
y quizas un poco mas de teora, pero en la mayora de los casos lo mas importante es tener
una idea para resolverlos.
2.1. Primera Etapa
1. En la siguiente gura AB = BC, AF = FD = DB y FD es paralela a AC.
Encuentra el valor del angulo CFD.
(a) 15

(b) 20

(c) 30

(d) 45

2. Pedro gana 10000 pesos menos que el doble de lo que gana Juan y Juan gana 22000
pesos menos del triple de lo que gana Pedro. Cuanto dinero ganan entre los dos?
(a) 10400 (b) 10800 (c) 21200 (d) 32000
7
8 CAP

ITULO 2. ENUNCIADOS DE LOS PROBLEMAS


3. Cuantas parejas (a, b) hacen al n umero 23a219b2 m ultiplo de 72?
(a) 2 (b) 3 (c) 4 (d) 6
4. A un tablero de 20 las y 11 columnas se le quitan las las pares y las columnas que
no son m ultiplo de 3. Cuantas casillas quedan?
(a) 140 (b) 90 (c) 50 (d) 30
5. Cuantos triangulos existen tales que las longitudes de sus lados son tres n umeros
distintos de la sucesion 1, 2, 4, 8, 16, . . . ?
(a) 0 (b) 1 (c) Una innidad (d) Ninguna de las anteriores
6. Un gato comienza en la orilla A de una viga AB de longitud 120. Comienza a caminar
sobre la viga y al llegar a la mitad otro gato comienza a caminar desde la orilla B a
la mitad de la velocidad que el primer gato. Si se encuentran en el punto C, cual
es el valor del producto AC CB?
(a) 1800 (b) 2000 (c) 2700 (d) 3200
7. En un salon el porcentaje de mujeres es mayor al 40 % y menor al 45 %, cual es la
mnima cantidad de personas que tiene que haber en el salon?
(a) 9 (b) 8 (c) 5 (d) Ninguna de las anteriores
8. En la gura se tienen dos crcunferencias concentricas. La cuerda AB de la circun-
ferencia mas grande es tangente a la circunferencia peque na y mide

3. Encuentra
el area entre las dos circunferencias.
(a) 3 (b)
3
2
(c)

3
2
(d)
3
4
9. Cual es el valor de la expresion 56335579
2
(56335591)(56335567)?
(a) 44 (b) 64 (c) 144 (d) 2010
2.1. PRIMERA ETAPA 9
10. Gogo tiene 4 amigas. Para da de San Valentn compro 3 peluches iguales y 2 rosas
iguales, de cuantas formas puede obsequiar todos sus regalos si los puede obsequiar
como quiera?
(a) 4
5
(b) 5
4
(c) 200 (d) 24
11. Cuantas semanas son 12! segundos? Recuerda que n! = 1 2 n es el producto
de los enteros de 1 a n.
(a) 6! (b) 792 (c) 704 (d) 396
12. Cual es el maximo n umero de crculos de radio 1 que pueden ser tangentes a un
crculo de radio 1 al mismo tiempo?
(a) 4 (b) 5 (c) 6 (d) 8
13. Tres alegras, tres cocadas y dos obleas cuestan 20 pesos. Cinco alegras, cinco coca-
das y tres obleas cuestan 33 pesos. Cuanto cuestan una alegra, una cocada y una
oblea?
(a) 7 (b) 8 (c) 9 (d) No se puede determinar
14. Se escriben los n umeros del 1 al 2010. Cuantas veces se escribio el dgito 2?
(a) 591 (b) 592 (c) 611 (d) 612
15. En el siguiente dibujo AB = BB

, BC = CC

, CD = DD

y DA = AA

. Encuentra
la razon entre las areas de los cuadrilateros ABCD y A

.
(a)
1
2
(b)
1
4
(c)
1
5
(d) Depende del cuadrilatero ABCD
16. Cual es la suma de los divisores impares de 2010?
(a) 75 (b) 1632 (c) 2458 (d) 4896
10 CAP

ITULO 2. ENUNCIADOS DE LOS PROBLEMAS


17. Cuanto vale x en la siguiente ecuacion?
10
x
= (10
2010
+ 25)
2
(10
2010
25)
2
(a) 2010 (b) 2012 (c) 4020 (d) 4024
18. En una urna hay 7 pelotas rojas, 8 pelotas azules y 9 pelotas verdes. Si se sacan
pelotas sin ver y sin regresarlas, cual es el mnimo n umero de pelotas que tienes
que sacar para garantizar que sacaste al menos dos de cada color?
(a) 6 (b) 17 (c) 19 (d) 24
19. Cual es el valor del area sombreada en el siguiente rectangulo?
(a) 32 (b) 38 (c) 54 (d) 70
20. Cuantos n umeros n hay tales que n! termina en 30 ceros pero no en 31?
(a) 0 (b) 5 (c) 10 (d) Una innidad
21. Una cuadrcula rectangular tiene 1000 cuadritos. La razon de sus lados es aproxima-
damente 3 a 5. Cuantos cuadritos hay en la orilla de la cuadrcula?
(a) 130 (b) 128 (c) 126 (d) 124
22. Cuantos palillos de longitud 1 se necesitan para hacer una cuadrcula de 6 6 de
longitud de lado 6 con sus lneas interiores?
(a) 36 (b) 42 (c) 72 (d) 84
23. En un triangulo un angulo mide 50

y otro x. Si sabemos que es isosceles, cual es


la suma de los tres posibles valores de x?
(a) 195

(b) 180

(c) 165

(d) 130

2.1. PRIMERA ETAPA 11


24. Pedro y Luis copiaron una ecuacion x
2
+bx+c = 0 de grado dos del pizarron. Pedro
copio mal el coeciente b y obtuvo como soluciones 2 y 4. Luis copio mal el coeciente
c y obtuvo como soluciones 5 y 4. Cuales eran las soluciones de la ecuacion original?
(a) 2 y 5 (b) 4 y 4 (c) 1 y 8 (d) 5 y 8
25. Un n umero es casi-capic ua si al cambiar uno de sus dgitos se lee igual de derecha
a izquierda que de izquierda a derecha. cuantos n umeros casi-capic ua hay de 4
dgitos?
(a) 3510 (b) 3330 (c) 3150 (d) 1620
26. Se tienen 3 circunferencias concentricas, la mas peque na tangente a un cuadrado y la
mediana circunscrita a ese cuadrado como se muestra en la gura. La circunferencia
menor tiene radio 1 y la mayor tiene diametro 6. Cual es el valor del area del anillo
formado entre la circunferencia mediana y la mayor?
(a) 7 (b) 8 (c) 34 (d) 35
27. De las fracciones de la forma
a
2010
con 0 < a < 2010 entero, cuantos denominadores
distintos pueden tener una vez que se simplican?
(a) 7 (b) 15 (c) 16 (d) 2009
28. Cuando el panda Gomibola desperto tena 5 % de su peso en leche. Tras desayunar
leche, su peso total subio a 3,8 kilos y el porcentaje de leche en su cuerpo aumento a
7 %. Cuanto pesaba el panda al despertar?
(a) 3.534 (b) 3.720 (c) 176.7 (d) 190
29. Se escriben de menor a mayor los n umeros que tienen exactamente una vez a cada
uno de los dgitos de 1 a 9. Que n umero queda en la posicion 100000?
(a) 367245918 (b) 358926471 (c) 235497681 (d) 249873561
12 CAP

ITULO 2. ENUNCIADOS DE LOS PROBLEMAS


30. Se toma un punto P en el interior del cuadrado ABCD. Si se sabe que AP = 3,
DP = 4 y CP = 7, cual es la razon
PA
PB
?
(a)

42
14
(b)
1
2
(c)
4
7
(d)

42
2.2. SEGUNDA ETAPA 13
2.2. Segunda Etapa
1. Se dobla una hoja cuadrada a la mitad para formar un rectangulo, luego a la mitad
para formar un cuadrado y una tercera vez para formar un triangulo. Cuantos de
los siguientes dibujos pueden representar como se ven los dobleces en la hoja cuando
se desdobla?
(a) 1 (b) 2 (c) 3 (d) 4
2. Los enteros a y b son mayores a 1 y tienen producto 2010. Si a es par, cual es su
mayor valor posible?
(a) 2 (b) 402 (c) 2010 (d) Ninguna de las anteriores
3. Se tiene un contenedor con forma de cubo de 30cm de lado. Si se le vertieran 14,4
litros de agua, a que altura llegara el agua?
(a) 1.4cm (b) 1.6cm (c) 14cm (d) 16cm
4. Se tienen dos pistas para correr cuadradas, una de lado 150m y otra de lado 100m.
Se quiere correr algunas vueltas completas a la primera y algunas vueltas completas
a la segunda para recorrer un total de 5,4km. Cual es el maximo n umero de vueltas
que se pueden dar si se quiere cumplir estas condiciones?
(a) 11 (b) 12 (c) 13 (d) 14
5. La siguiente casa esta formada por cuatro cuadrados iguales y un triangulo equilate-
ro. Si el permetro de la gura es 30, cual es el area de la region sombreada?
(a) 4 (b)
17
4
(c)
75
12
(d) 9
14 CAP

ITULO 2. ENUNCIADOS DE LOS PROBLEMAS


6. El dgito de las unidades de 3
17
+ 7
13
es:
(a) 0 (b) 2 (c) 4 (d) 6
7. Cuantos rectangulos hay en el siguiente dibujo de una cancha de tenis?
(a) 23 (b) 29 (c) 30 (d) 31
8. En la siguiente gura se tiene un paralelogramo. Encuentra la razon entre el area de
la region negra y el area de la region blanca.
(a)
1
2
(b) 1 (c)
2
3
(d) No se puede determinar
9. La suma de 20 n umeros enteros es 200. De estos, cual es la mayor cantidad de ellos
que puede ser mayor a 20?
(a) 20 (b) 19 (c) 10 (d) 9
10. En el triangulo ABC se toman los puntos medios D y E de BC y AC respectiva-
mente. Si el area del triangulo ADE es 16, cual es el area del triangulo ABC?
(a) 48 (b) 64 (c) 80 (d) No se puede determinar sin conocer al triangulo ABC
2.2. SEGUNDA ETAPA 15
11. Un a no humano equivale a 7 a nos gato o a 6 a nos perro. Hoy el gato Miau cumple
21 a nos gato y el perro Guau cumple 18 a nos perro. Cual es la diferencia entre la
edad perruna de Miau y la edad gatuna de Guau?
(a) 0 (b)
13
14
(c) 3 (d)
127
14
12. En el cuadrado ABCD de permetro 40 se ha elegido un punto E sobre la diagonal
BD de modo que 4DE = BE. Cual es el valor del area del triangulo AED?
(a) 4 (b) 10 (c) 12.5 (d) 20
13. Cual es la suma de los primeros 100 terminos de la sucesion
1, 2, 1, 2, 2, 1, 2, 2, 2, 1, 2, 2, 2, 2, 1, 2, 2, 2, 2, 2, 1, . . .?
(a) 113 (b) 150 (c) 157 (d) 187
14. En el triangulo PQR se tiene PQ = 14 y PR = 10. El lado RQ intersecta perpen-
dicularmente a PS, con RS = 5, cual es el permetro del triangulo PQR?
(a) 15 + 5

3 (b) 25 + 5

3 (c) 30 (d) 35
15. Sea P(k) la suma de los dgitos pares de k. Por ejemplo, P(5681) = 6 + 8 = 14.
Cual es el valor de P(1) +P(2) +. . . +P(100)?
(a) 400 (b) 250 (c) 200 (d) 100
16 CAP

ITULO 2. ENUNCIADOS DE LOS PROBLEMAS


16. En la siguiente suma cada letra representa un dgito y el punto es un punto decimal:
abc+ab.c+a.bc = k. Si se quiere que k sea entero, cual de las siguientes armaciones
es verdadera?
(a) a = b = c
(b) 37 divide a k
(c) a +b +c es constante
(d) Hay exactamente 5 valores posibles para k
17. Las cuerdas PR y QS de la siguiente circunferencia se interectan en el punto T. Si
el angulo PTQ mide 20

y el radio del crculo es 5, cuanto vale la suma de las


longitudes de los arcos RS y PQ?
(a)
5
9
(b)
10
9
(c)
5
4
(d)
9
5
18. Se llena una tabla compuesta por p las y q columnas con los n umeros naturales del
1 a pq escritos en orden, empezando en la esquina superior izquierda, avanzando a
la derecha y luego cambiando de la, volviendo a empezar a la izquierda. El n umero
20 se encuentra en la tercer la, el n umero 41 en la quinta y el 103 en la ultima.
Encuentra el valor de p +q
(a) 21 (b) 22 (c) 23 (d) 24
19. Se tiran dos dados negros y uno rojo. Cual es la probabilidad de que el n umero en
el dado rojo sea mayor a los dos n umeros de los dados negros?
(a)
225
1296
(b)
55
216
(c)
1
3
(d)
91
216
2.2. SEGUNDA ETAPA 17
20. Los triangulos ABC y ABD de la siguiente gura son isosceles con AB = AC = BD.
Si BD es perpendicular a AC, cuanto vale BCA+BDA?
(a) 120

(b) 135

(c) 145

(d) No se puede determinar


21. La Olimpiada de Matematicas es un evento anual. Este a no se lleva a cabo la edicion
n umero 24. Un a no se dice superolmpico si la edicion del concurso divide al a no en
que se realiza (por ejemplo, 2010 no es un a no superolmpico pues 24 no divide a
2010). Cuantos a nos superolmpicos existen?
22. Un cohete viaja de la Tierra a la Luna. Si usa el propulsor A, llega en 200 horas. Si
usa el propulsor A y el propulsor B llega en 150 horas y si usa el propulsor A y el
propulsor C llega en 120 horas. En cuanto tiempo llega si utiliza el propulsor B y
el propulsor C?
23. Cuantos n umeros de cuatro dgitos tienen a lo mas un dgito 6?
24. Los puntos medios de los lados de un hexagono regular H se unen para obtener un
hexagono similar H

. Que fraccion del area de H es el area que se encuentra entre


H y H

?
25. Sabemos que 24! = 620448401733239439abc000 donde a, b y c representan dgitos.
Escribe el n umero de tres dgitos abc.
18 CAP

ITULO 2. ENUNCIADOS DE LOS PROBLEMAS


2.3. Tercera Etapa
Problema 1 Sea ABC un triangulo con AD la altura sobre BC. La circunferencia
de centro D y radio AD corta a AB en P y a AC en Q. Demuestra que los triangulos
ABC y AQP son semejantes.
Problema 2 Fer, Gogo, Leo y Marco compran una barra de chocolate rectangular
de mn cuadritos unitarios. Van a jugar a lo siguiente sobre una mesa, alternandose de
la forma F, G, L, M, F, G, . . ., etc.
En el primer turno Fer toma el chocolate y lo corta en lnea recta por una de las
lneas que dividen a los cuadritos. En los turnos siguientes, de las piezas de chocolate en
la mesa el jugador correspondiente toma una de ellas (que no sea un cuadrito individual)
y hace lo mismo: cortarla por una lnea recta que pase por las divisiones de cuadritos.
Gana el jugador que al nalizar su turno deje la mesa con los mn cuadritos unitarios. A
continuacion se muestra un ejemplo del juego para m = 2 y n = 3, en el cual gana Fer:
Dados los lados m y n del chocolate, determina que jugador puede ganar.
Problema 3 Sea n un entero mayor o igual a 10. A n se le quita el dgito de las
unidades y se obtiene un n umero m. Encuentra todos los n tales que m divide a n.
Problema 4 Pichi escribe todos los n umeros mayores a 10000 que se pueden formar
con dgitos a, b, c, d y e (no necesariamente en ese orden) que cumplen la condicion de
que b = a + 2 , c = b + 2, d = c + 2 y e = d + 2.
Cuantos n umeros escribio Pichi?
Calcula la suma de los n umeros que escribio Pichi.
2.4. CUARTA ETAPA 19
2.4. Cuarta Etapa
Primer Da
Problema 1 Un n umero natural de n dgitos es armonioso si sus n dgitos son
una permutacion de 1, 2, 3, . . . , n y sus primeros k dgitos forman un n umero divisible por
k, para k = 1, 2, 3, . . . , n. Por ejemplo, 321 es armonioso pues 3 es divisible por 1, 32 es
divisible por 2 y 321 es divisible por 3. Encuentra todos los n umeros armoniosos de 6
dgitos.
Problema 2 Sea ABC un triangulo y sean D y E puntos sobre los segmentos AB
y AC tales que BD+CE = BC. Si I es el incentro del triangulo ABC, demuestra que el
cuadrilatero ADIE es cclico.
Problema 3 Demuestra que:
a)
1
3

1
2
+
3

12+
3

2
2
+
1
3

3
2
+
3

34+
3

4
2
+. . . +
1
3

999
2
+
3

9991000+
3

1000
2
>
9
2
b)
1
3

2
2
+
3

23+
3

3
2
+
1
3

4
2
+
3

45+
3

5
2
+. . . +
1
3

998
2
+
3

998999+
3

999
2
<
9
2
Segundo Da
Problema 4 La FIFA desea cambiar la modalidad del mundial. En el torneo par-
ticiparan 32 equipos, los juegos de cada ronda se decidiran por sorteo y en cada partido
entre dos equipos exactamente uno, el ganador (no hay empates), pasara a la siguiente
ronda. La FIFA tiene un ranking de los 32 equipos ordenandolos de mejor a peor. En la
primera ronda del torneo se realizaran 16 partidos y los 16 ganadores pasan a la segunda
ronda, en la segunda ronda se jugaran 8 partidos y los 8 equipos ganadores pasan a la
tercera ronda as hasta que en la cuarta ronda habra 2 partidos y los ganadores jugaran
la nal. Supongamos que si un equipo A esta en mejor posicion en el ranking de la FIFA
que un equipo B entonces si A y B juegan, A le gana a B, por ejemplo el equipo 1 en el
ranking siempre gana. Bajo esta suposicion, cual es el peor equipo que puede disputar la
nal?
Problema 5 Sea ABC un triangulo con AB = AC y A = 36

. La bisectriz del
ABC corta a AC en D y la bisectriz del BDC corta BC en P. Sea R un punto sobre
la prolongacion de BC tal que B es el punto medio de PR. Demuestra que RD = AP.
Problema 6 Halla todos los enteros n > 1, tales que cada divisor primo de n
6
1
es divisor de n
2
1 o de n
3
1.
20 CAP

ITULO 2. ENUNCIADOS DE LOS PROBLEMAS


2.5. Quinta Etapa
Primer Examen, Primer Da
Problema 1 Se tiene el n umero 1234 escrito en una hoja blanca. Cesar y Lalo
toman turnos para jugar lo siguiente: en cada turno un jugador puede restarle cualquiera
de los dgitos del n umero escrito en la hoja (siempre y cuando este dgito no sea cero);
se borra el viejo n umero y se escribe el nuevo. Pierde el que ya no pueda restar n umeros
(cuando se llegue al n umero cero). Si Cesar es el que empieza, quien gana y cual es su
estrategia ganadora?
Problema 2 Sea ABC un triangulo rectangulo con angulo recto en B. Sea E el pie
de la altura desde B y consideremos I
A
e I
C
los incentros de ABE y CBE respectivamente.
Demuestra que AI
A
es perpendicular a BI
C
.
Demuestra que la bisectriz por B es perpendicular a I
A
I
C
.
Problema 3 Sean a, b y c n umeros reales mayores que cero. Demostrar que:
a
b
+
b
c
+
c
a

_
b
a
_
3
+
_
c
b
_
3
+
_
a
c
_
3
Primer Examen, Segundo Da
Problema 4 Determinar todos los n umeros primos p tales que 5
p
+ 4p
4
es un
cuadrado perfecto.
Problema 5 Sea ABC un triangulo y G su gravicentro. Se construyen triangulos
equilateros exteriores BCD, CAE y ABF con gravicentros A

, B

y C

. Demuestra que
G es el circuncentro del A

.
Problema 6 Lalo y Cesar juegan volados. Lanzan n veces la moneda. Cesar gana
si la cantidad de aguilas obtenidas es m ultiplo de 4, y Lalo gana en otro caso. Para que n
la probabilidad de que gane Cesar es
1
4
?
2.5. QUINTA ETAPA 21
Segundo Examen, Primer Da
Problema 1 Encuentra todos los enteros positivos n tales que 3n 4, 4n 5 y
5n 3 sean simultaneamente primos.
Problema 2 Sea a
1
, a
2
, a
3
, . . ., una sucesion de n umeros de tal manera que a
1
= 1
y para n > 1, a
1
+a
2
+. . . +a
n
= n
2
a
n
. Encuentra a
2010
.
Problema 3 Sea M el punto medio del segmento AB. Dos circunferencias, una que
pasa por A y M, y otra que pasa por B y M, se intersectan en M y N. Sean P el punto
medio del arco AN que no contiene a M en el circuncrculo del triangulo MAN y Q el
punto medio del arco BN que no contiene a M en el circuncrculo del triangulo MBN.
Demuestra que las rectas MN y PQ son perpendiculares entre s.
Segundo Examen, Segundo Da
Problema 4 Un triangulo ABC es tal que su angulo en A es de 60

. Sean D y E
puntos sobre los lados AB y AC respectivamente de manera que BD = DE = EC. Sea O
el punto de interseccion de BE y DC. Demuestra que O es el circuncentro del triangulo
ABC.
Problema 5 Un marciano esta atrapado en un cuadrito de 11 de una cuadrcula de
nn donde n es un entero positivo impar mayor que 1. Se ha enviado una nave a rescatarlo,
pero la nave no logra ver al marciano. La nave puede teletransportar al marciano abordo
con disparos de rescate, cada uno de ellos cubre tres cuadros de la siguiente forma:
Los disparos se pueden encimar, rotar y salir de la cuadrcula. Cual es el mnimo
n umero de disparos de rescate que tiene que hacer la nave para asegurarse de haber
rescatado al marciano?
Problema 6 Determina todas las parejas (a, b) de enteros positivos tales que ab
2
+9
divide a a
2
b +b.
22 CAP

ITULO 2. ENUNCIADOS DE LOS PROBLEMAS


2.6. Examen Regional
Problema 1 En el triangulo acutangulo ABC se tiene que BAC es menor que
ACB. Sea AD un diametro de , la circunferencia circunscrita a dicho triangulo. Sea E
el punto de interseccion del rayo AC y la tangente a que pasa por B. La perpendicular
a AD que pasa por E intersecta a la cirunferencia circunscrita del triangulo BCE, otra
vez, en el punto F. Muestra que CD es bisectriz del BCF.
Problema 2 Sea p > 5 un n umero primo. Muestra que p 4 no puede ser la cuarta
potencia de un entero.
Problema 3 Sean a, b, c n umeros reales positivos que cumplan con
1
a
+
1
b
+
1
c
= 1.
Muestra que:
a
2
+b
2
+c
2
2a + 2b + 2c + 9
Problema 4 Sean a y b dos enteros positivos. Sea A un subconjunto de 1, 2, . . . , a+
b con mas de
a+b
2
elementos. Muestra que hay dos n umeros en A cuya diferencia es a o
b.
Problema 5 Encuentra las soluciones enteras (p, q, r) de la ecuacion r + p
4
= q
4
con las siguientes condiciones:
r es un entero positivo con exactamente 8 divisores positivos.
p y q son n umeros primos.
Problema 6 Sean ABC un triangulo equilatero y D el punto medio de BC. Sean
E y F puntos sobre AC y AB, respectivamente y tales que AF = CE. Si P es el punto
de interseccion de BE y CF, muestra que APF = BPD.
2.7. EXAMEN DE DESEMPATE 23
2.7. Examen de Desempate
Problema 1 Sea ABCD un cudrlatero cclico con diagonales perpendiculares. Sea
E el punto de interseccion de la perpendicular a AB por B y la paralela a BD por C y
sea F el punto de interseccion de AE y BD. Demuestra que los triangulos AFD y ABC
son semejantes.
Problema 2 Sean a, b y p enteros no negativos con p un n umero primo. Encontrar
todas las ternas (a, b, p) tales que a
2
b
2
= 2
2010
pb.
Problema 3 Toma k entero positivo jo. Los vertices de un nagono regular se
quieren pintar con k colores distintos de modo que cada que los vertices A
1
, A
2
y A
3
estan
pintados del mismo color y los vertices B
1
, B
2
y B
3
estan pintados del mismo color (quizas
distinto al de los vertices A
i
), entonces los triangulos A
1
A
2
A
3
y B
1
B
2
B
3
comparten al
menos un punto (en su interior, lados, o vertices). Cual es el mayor valor que puede tomar
n?
24 CAP

ITULO 2. ENUNCIADOS DE LOS PROBLEMAS


Captulo 3
Soluciones de los Problemas
3.1. Primera Etapa
1. Por el Teorema de Tales, tenemos
BF
BA
=
BD
BC
. Como BA = BC, esto nos dice
BF = BD. Tenemos tambien BF = AB AF = BC BD = DC. As, BF =
FA = FD = BD = DC. De esto deducimos que el triangulo BFD es equilatero y
FDC isosceles. As, 2DFC = DFC +DCF = 180

FDC = BDF = 60

,
y por tanto DFC = 30

.
2. Llamemos j a la cantidad que gana Juan y p a la cantidad que gana Pedro. El
enunciado dice que p = 2j10000 y j = 3p22000. Multiplicando la primer ecuacion
por 3 y subsituyendo en la segunda obtenemos j = 3p22000 = 6j 3000022000.
De aqu despejamos para obtener 5j = 52000, de modo que j = 10400. Substituyendo
el la segunda ecuacion, 3p = 22000+10400 = 32400 y as p = 10800. Sumando ambos
ingresos obtenemos 21200
3. Como 72 = 8 9 y 8 y 9 son primos relativos, 72 divide a un n umero si y solo si
8 y 9 lo dividen. Para que un n umero sea divisible entre 8, sus tres ultimos dgitos
deben ser divisibles entre 8. Notemos que 992 es divisible entre 8. As, el n umero
que buscamos puede acabar en 992, 952 o 912 (pues estos n umeros dieren en 40,
un m ultiplo de 8), de modo que b puede ser 1, 5 o 9.
Para que un n umero sea m ultiplo de 9, la suma de sus dgitos debe ser m ultiplo de
9. Si b = 1, la suma de los dgitos es 20 +a y entonces a necesita ser 7. Si b = 5, la
suma es 24 + a y a necesita ser 3. Finalmente, si b = 9, la suma es 28 y a necesita
ser 8. As, en total hay 3 posibilidades para a y b.
4. Primer solucion Originalmente hay 220 cuadritos. Quitando las las pares, se quitan
10 11 = 110 cuadritos. Quitando las columnas que no son m ultiplo de 3, se quitan
20 8 = 160 cuadritos. Sin embargo, los 10 9 = 90 cuadritos que estan en la impar
y columna no m ultiplo de 3 los quitamos 2 veces, as que hay que regresarlos. De
esta forma, en total quedan 220 110 160 + 90 = 40 cuadritos.
25
26 CAP

ITULO 3. SOLUCIONES DE LOS PROBLEMAS


Segunda solucion Hay 10 n umeros de 1 a 20 que son impares y 4 n umeros de 1 a
11 que s son m ultiplos de 3, y un cuadrito queda si y solo si no lo quitamos con
ninguno de los dos movimientos. De esta forma, quedan 4 10 = 40 cuadritos.
5. No hay ning un triangulo que se pueda hacer. Tomemos tres n umeros 2
n
, 2
m
y 2
l
con
n < m < l. Tenemos que 2
n
+ 2
m
< 2
m
+ 2
m
= 2
m+1
2
l
as, 2
n
+ 2
m
< 2
l
y por
tanto no se cumple la desigualdad del triangulo.
6. Llamemos M al punto medio de AB. Como el primer gato va al doble de velocidad,
MC
CB
= 2. As,
BM
BC
=
BC+CM
BC
= 1 +
CM
BC
= 3. Con esto obtenemos
CB
AB
=
CB
2BM
=
1
6
,
por lo que CB mide un sexto de 120, es decir, mide 20. Esto nos dice tambi!n que
AC mide 100. As, AC CB = 2000.
7. La respuesta es ninguna de las anteriores. Si hubiera 7 personas, 4 de ellas podran
ser mujeres y el resto hombres y entonces aproximadamente el procentaje de mujeres
sera 42,8 %. Con 5 personas no es posible, pues los porcentajes consecutivos posibles
20 % y 40 % no estan en el rango pedido.
8. Denotemos por R al radio de la circunferencia mayor y por r al radio de la circun-
ferencia menor. Llamemos ademas M al punto medio de AB y O al centro de las
circunferencias. Por simetra, tenemos que M es el punto de tangencia de AB. Por
esta razon, OM es perpendicular a AB. Usando el Teorema de Pitagoras obtenemos:
r
2
+MB
2
= R
2
. Asi, R
2
r
2
= MB
2
=
_

3
2
_
2
=
3
4
. El area del anillo esta dada por
la diferencia de area de las circunferencias, es decir: R
2
r
2
= (R
2
r
2
) =
3
4
.
9. Hagamos x = 56335579. Desarrollando: 56335579
2
(56335591)(56335567) = x
2

(x + 12)(x 12) = x
2
(x
2
144) = 144.
10. Como la reparticion de las rosas y de los peluches son independientes, podemos
contar de cuantas formas podemos repartir rosas, luego peluches y multiplicar ambos
resultados.
Para repartir las rosas tenemos los siguientes casos:
Gogo le da las rosas a la misma amiga. En este caso hay 4 opciones, una por
amiga.
Gogo le da las rosas a amigas distintas. En esta caso hay
_
4
2
_
= 6 formas de
elegir las 2 amigas a quienes le dara las rosas.
En total hay 10 opciones para la reparticion de rosas. Seguimos un analisis similar
con la reparticion de peluches:
Gogo le da los peluches a la misma amiga. En este caso hay 4 opciones, una
por amiga.
Gogo le da 2 peluches a una amiga y 1 a otra. En este caso hay 4 3 = 12 formas
de elegir, primero elige a quien le va a dar 2 y luego entre las otras 3 elige a
quien le va a dar 1.
3.1. PRIMERA ETAPA 27
Gogo le da los peluches a amigas distintas. En este caso solo tiene que elegir a
quien no le toca peluche, y hay 4 opciones para esto.
En total hay 20 opciones para la reparticion de rosas. Por cada forma de repartir
rosas hay una para repartir peluches, y por tanto en total tenemos 20 10 = 200
formas para repartir los regalos.
11. Haciendo las conversiones necesarias, y cancelando para no efectuar tantas opera-
ciones, tenemos:
12! seg = 12! seg
1 min
60 seg

1 hr
60 min

1 da
24 hr

1 sem
7 da
=
12 11 10 9 8 7 6 5 4 3 2 sem
60 60 24 7
= 2 4 9 11 sem = 792 sem
12. El siguiente dibujo muestra como se pueden colocar 6 circunferencias de radio 1
alrededor.
13. Denotemos por a, c y o el precio de las alegras, cocadas y obleas respectivamente.
La informacion del problema nos dice que 3a + 3c + 2o = 20 y 5a + 5c + 3o = 33.
Multiplicando la primer ecuacion por 2 y restandole la segunda, obtenemos que
a +c +o = 7.
14. Solucion 1 Escribamos a todos los n umeros menores a 1000 y mayores o iguales
a 0 como n umeros de 3 dgitos, por ejemplo, el 21 lo escribimos como 021 y el 0
como 000. En total habremos escrito 1000 3 = 3000 dgito, y cada dgito entre 0
y 9 aparece la misma cantidad de veces. Como hay 10 dgitos distintos, cada dgito
esta escrito 300 veces, en particular el 2. As mismo, entre 1000 y 1999 aparecen
otros 300 dgitos 2. Finalmente, de 2000 a 2010 aparecen 12 dgitos 2. Esto nos da
un total de 612 dgitos 2 de 0 a 2010, y hay la misma cantidad de 1 a 2010.
Solucion 2 El 2 aparece en las unidades cada 10 n umeros, dando un total de 201
dgitos 2. Aparecen 10 de ellos en las decenas cada 100 n umeros, dando un total de
otros 200. De la misma forma, cada 1000 n umeros hay 100 dgitos 2 en las centenas,
sumando otros 200. Finalmente, en los millares aparecen solo 11 veces. De nuevo,
juntamos 612 dgitos 2.
15. Denotaremos con parentesis el area de una gura. Como A es punto medio de A

D,
tenemos que (A

AB) = (ADB). As mismo, (A

BB

) = (A

AB). De este modo,


(AA

) = 2(ABD). Analogamente, (DD

) = 2(CAD), (CC

) = (BDC) y
(BB

) = 2(ABC). Sumando estas areas, mas la del cuadrilatero ABCD obtenemos


(A

) = (AA

) + (DD

) + (CC

) + (BB

) + (ABCD) = 2(ABC) +
2(BCD) +2(CDA) +2(DAB) +(ABCD) = 5(ABCD). As, la razon buscada es
1
5
.
28 CAP

ITULO 3. SOLUCIONES DE LOS PROBLEMAS


16. Los divisores impares de 2010 son los divisores de 1005. Como 1005 = 3 5 67,
sus divisores son 1,3, 5, 15, 67, 201, 335 y 1005. La suma de estos n umeros es 1632.
17. Como el lado derecho es una diferencia de cuadrados, tenemos:
(10
2010
+ 25)
2
(10
2010
25)
2
= (10
2010
+ 25 + 10
2010
25) (10
2010
+ 25 10
2010
+ 25)
= (2 10
2010
)(50) = 100 10
2010
= 10
2012
As, x es 2012.
18. Si sacamos 9 pelotas verdes, 8 azules y 1 roja ya sacamos 18 pelotas, pero a un no
tenemos 2 de cada color. Sin embargo, sin importar como saquemos 19 pelotas, al
menos 2 de ellas tienen que ser rojas (pues con las verdes y azules maximo juntamos
17) y analogamente al menos 2 de ellas seran azules y 2 de ellas verdes. As, la
respuesta es 19.
19. Llamemos O al centro del rectangulo, A al punto en el lado superior, B al punto
en la esquina superior derecha y C al punto sobre el lado derecho. Por simetra, O
es el centro del rectangulo. De modo que el triangulo OAB tiene base 16 y altura
2+8
2
= 5. As, tiene area 40. Del mismo modo, el triangulo OBC tiene base 2 y altura
16+12
2
= 14. As, tiene area 14. Sumando el area de estos triangulos obtenemos que
el area de OABC es 54.
20. Un n umero termina en tantos 0 como la cantidad de veces que tiene a 10 como factor.
As, si un n umero tiene n factores 2 y m < n factores 5, entonces terminara en m
ceros. El n umero n! entra en esta observacion, pues tiene mas factores 2 que factores
5. Cada 5 n umeros se aumenta un factor 5, cada 25 se aumenta uno mas, cada 125
otro mas y as. Contando los factores as, el n umero 120! termina en 29 ceros, pero
125! termina en 31 ceros. As, para ning un n umero tenemos que n! termina en 30
ceros.
21. Solucion 1 Las factorizaciones distintas de 1000 son 1 1000, 2 500, 4 250, 8 125,
5 200, 10 100, 20 50 y 25 40. La ultima es la que mas se acerca a la razon 3 a 5, pues
25
40

24
40
=
3
5
. La cantidad de cuadritos en dicha cuadrcula es 25+25+40+404 =
126 (pues repetimos las 4 esquinas).
Solucion 2 Un lado del cuadrado mide aproximadamente 3x y el otro 5x para alguna
x. Tenemos entonces 3x 5x 1000, o bien x
2

1000
15
66. As, x 8. Notamos que
los lados son aproximadamente 24 y 40. Para que quede de lado 1000 necesitamos
que un lado sea 25, lo cual es posible, y de nuevo tenemos 126 cuadritos en la orilla.
22. Solucion 1 A cada cuadrito de 11 asignemosle sus lados izquierdo y derecho. Como
hay 36 cuadritos de estos, llevamos 72 palillos. Pero faltan contar los 12 palillos para
formar el lado derecho y el lado inferior del cuadrado. Esto da un total de 84 palillos.
3.1. PRIMERA ETAPA 29
Solucion 2 Para formar una linea horizontal o vertical se necesitan 6 palillos. En
total hay 7 lneas verticales y 7 horizontales. As, necesitamos 6 14 = 84 palillos.
Solucion 3 Usaremos Teora de Gracas. Hay 4 puntos a los que llegan 2 palillos (las
esquinas), 20 puntos a los que llegan 3 palillos (los de los lados) y 25 puntos a los que
llegan 4 palillos (los interiores). As, la suma de todos los grados es 8+60+100 = 168.
Un teorema de Teora de Gracas nos dice que hay
168
2
= 84 palillos.
23. Supongamos que los angulos iguales del triangulo ABC son B y C. Tenemos los
siguientes tres casos:
B = 50

, C = 50

y entonces x = 180

100

= 80

.
B = 50

, C = x y entonces x = 50

.
B = x, C = x, y entonces 2x = 180

50

= 130

y por tanto x = 65

Por lo tanto, la suma de los valores de x es 195

.
24. Si un polinomio cuadratico x
2
+ bx + c tiene soluciones r y s, entonces se factoriza
como (xr)(xs). As, el polinomio que copio Pedro fue (x2)(x4) = x
2
6x+8
y el que copio Luis fue (x5)(x4) = x
2
9x+20. Pedro tiene bien el coeciente c y
Luis el coeciente b, de modo que el polinomio original era x
2
9x+8. Resolviendolo
(factorizando o por formula general), obtenemos que sus races son 8 y 1.
25. Denotaremos dgitos con letras. Un n umero casi-capic ua es de alguna de las siguientes
dos formas:
abbc. En este caso tenemos 9 opciones para a, pues no puede ser 0. El dgito
c debe ser distinto a a, por lo que tenemos 9 opciones. Finalmente, el dgito b
puede ser cualquiera, de modo que hay 810 casi-capic uas de estos.
abca. Tambien tenemos 9 opciones para a. Al elegir b entre los 10 dgitos, c deber
ser distinto y por tanto tiene 9 opciones. De nuevo, juntamos 810 casi-capic uas.
En total tenemos 1620 casi-capic uas de 4 dgitos.
26. El dametro de la circunferencia menor es 2 y por tanto el lado del cuadrado es
2. Con el Teorema de Pitagoras obtenemos que la diagonal del cuadrado d cumple
d
2
= 2
2
+2
2
= 8. As, d = 2

2 y por tanto el radio de la circunferencia mediana es la


mitad,

2. Ademas, el radio de la mayor es 3 As, el area del anillo es 3


2
sqrt2
2
=
9 2 = 7.
27. Hay tantos denominadores como divisores tenga 2010, excepto que no podemos poner
a = 2010. Como 2010 = 2 3 5 67, entonces 2010 tiene 16 divisores. Quitando a
2010, tenemos 15 posibles numeradores.
28. Tras desayunar, los datos nos dicen que de los 3,8 kilos que pesa tras desayunar,
el 7 % es leche, de modo que tiene 0,266 kilos de leche. Como solo desayuno leche,
30 CAP

ITULO 3. SOLUCIONES DE LOS PROBLEMAS


la diferencia de peso total es la misma que la diferencia de peso en leche. Ademas,
si el peso inicial es p, el peso inicial en leche es 0,05p. Esto nos da la ecuacion
3,8 p = 0,266 0,05p. Resolviendo la ecuacion obtenemos p =
3,534
0,95
= 3,72.
29. Tenemos que cada 8! n umeros cambia el primer dgito (de izquierda a derecha),
cada 7! cambia el segundo, y as, hasta que cada n umero cambia el ultimo dgito.
Observemos que 100000 = 2 8! + 3 7! + 5 6! + 5 5! + 4! + 2 3! + 2 2!. As, el
primer dgito queda jo en 3, el segundo en 5 (cambia tres veces, pero ya no puede
ser 2) y si seguimos as, llegamos al n umero 358926471.
30. Tracemos la paralela a AD que pasa por P. Supongamos que corta a AB en X y a
CD en Y . Como ABCD es cuadrado, XY tambien es paralela a BC, AX = DY y
BC = CY . Usando repetidas veces el Teorema de Pitagoras tenemos: PB
2
BX
2
=
PX
2
= AP
2
AX
2
. Analogamente, PC
2
CY
2
= PD
2
DY
2
. As, manipulando
un poco estas igualdades y combinandolas obtenemos PB
2
PA
2
= BX
2
AX
2
=
CY
2
DY
2
= CP
2
DP
2
. Substituyendo por los valores conocidoes de AP, CP y
DP obtenemos: PB
2
= CP
2
+AP
2
DP
2
= 49+916 = 42. As, la razon buscada
es
3

42
. Racionalizando el denominador, obtenemos

42
12
.
3.2. SEGUNDA ETAPA 31
3.2. Segunda Etapa
1. Los primeros dos dobleces marcan una lnea horizontal y una lnea vertical en la
hoja. El tercer doblez tiene dos casos: pasar por la esquina del cuadrado original o
no. Estos casos corresponden respectivamente a los dibujos 1 y 4.
2. Los divisores pares de 2010 son 2, 6, 134, 10, 30, 670, 402 y 2010. Como b > 1,
necesitamos a < 2010. El mayor de los divisores que cumple esto es 670, por lo que
ninguna de las opciones a), b) o c) son correctas.
3. Un decmetro c ubico de agua es un litro. As, 14,4 litros son 14,4 decmetros c ubicos,
o bien 14400 centmetros c ubicos. El cubo tiene 900 centmetros cuadrados de base,
as que la altura a la que llega el agua es
14400
900
= 16 centmetros.
4. El recorrer la pista de lado 150 se recorren 600 metros. Al recorrer la de lado 100 se
recorren 400 metros. Para maximizar el n umero de vueltas con una cantidad ja a
correr, se necesita maximizar el n umero de vueltas a la pista chica. Darle 14 vueltas
ya se pasa. Dar 13 vueltas no se puede continuar con vueltas grandes. Dando 12
vueltas a la pista chica y 1 vuelta a la pista grande maximizamos el n umero de
vueltas chicas, dando un total de 13.
5. Denotemos el lado de cada cuadrado por L. Como el triangulo es equilatero, cada
uno de sus lados mide 2L. As, el permetro de la gura es 12L. Pero por hipotesis,
tambien es 30. De este modo L =
30
12
y por tanto el area del cuadrado es L
2
=
900
144
=
7512.
6. Al ver los dgitos de las unidades de las potencias de 3, vemos que se ciclan 3, 9, 7,
1, de modo que 3
17
termina en 3. Las de las potencias de 7 se ciclan 7, 9, 3, 1, de
modo que 7
13
termina en 7. De modo que su suma tiene a 0 como dgito nal.
7. Veamos que hay 6 rectangulos que tienen a la esquina superior izquierda como su
propia esquina superior izquierda. Fijandonos en los distintos puntos que hay y
contando cuantos rectangulos lo tienen como su propia esquina superior izquierda,
obtenemos 6, 6, 5, 3, 3, 2, 2, 1, 1, y 1 rectangulos, sumando un total de 30 rectangulos.
8. Solucion 1 Denotemos por a la base de un triangulo negro, como b la base del otro
y como h la altura del paralelogramo y por tanto de cada triangulo. El area del
paralelogramo esta dada por (a + b)h. El area de un triangulo es
ah
2
y del otro
bh
2
.
La suma de estas dos areas es
ah
2
+
bh
2
=
(a+b)h
2
. As, el area negra tiene la mitad
del area del paralelogramo y por tanto el area blanca tambien. Esto nos dice que la
razon entre ambas es 1.
Solucion 2 Si un triangulo tiene base en un lado de un paralelogramo y el otro
vertice se mueve en el lado opuesto del paralelogramo, entonces su area no cambia.
As, podemos mover ambos vertices superiores de los triangulos negros a la esquina
superior izquierda sin cambiar el area de la region negra. Claramente el area de la
32 CAP

ITULO 3. SOLUCIONES DE LOS PROBLEMAS


region que queda es la mitad del area del paralelogramo y por tanto la razon entre
el area blanca y la negra es 1.
9. Si todos ellos son mayores a 20, entonces la suma es al menos 400 y por tanto no
cumple. Sin embargo, si 19 de ellos son, por ejemplo 30, entonces al tomar el vigesimo
igual a 370 y entonces la suma de todos es 200. As, hasta 19 n umeros pueden ser
mayores a 20.
10. El triangulo ADC tiene la misma altura que el triangulo ABC (en el vertice A),
pero la mitad de lado, de modo que tiene la mitad del area de ABC. Del mismo
modo, el triangulo ADE tiene la misma altura que el triangulo ADC (en el vertice
D), pero la mitad de lado, de modo que tiene la mitad del area de ADC. As, el
triangulo ADE tiene un cuarto del area del triangulo ABC, y por tanto el area del
triangulo ABC es 4 16 = 64.
11. Miau cumple 21 a nos gato, o bien 3 a nos humano o bien, a 18 a nos perro. Guau
cumple 18 a nos perro, o bien 3 a nos humano, o bien 21 a nos gato. As, la diferencia
entre las edades que se piden es 3.
12. El cuadrado tiene permetro 40 y por tanto lado 10 y area 100. El triangulo ABD
tiene la mitad de dicha area, es decir, 50. Ahora, el triangulo AED comparte la
altura desde A con el triangulo ABD. Pero como 4DE = BE, entonces BD = 5DE,
entonces el triangulo AED tiene un quinto de area que el triangulo ABD, de modo
que es de area 10.
13. Los unos aparecen en las posiciones 1, 3, 6, 10 y en general en las posiciones de la
forma 1 +2 +. . . +n =
n(n+1)
2
. El n umero mas cercano de esta forma que no se pasa
de 100 es
1314
2
= 91. Hasta este momento se escribieron 13 unos y 9113 = 78 doses.
Ademas de esto, se escriben 9 doses mas. As, la suma buscada es 13+(78+9)2 = 187.
14. Por el Teorema de Pitagoras aplicado en los triangulos rectangulos PSR y PSQ,
tenemos que SP
2
= 100 25 = 75 y que SQ
2
= 14
2
SP
2
= 196 75 = 121. As,
SQ = 11 y por tanto RQ = SQSR = 11 5 = 6. De esta forma, el permetro del
triangulo PQR es 6 + 14 + 10 = 30.
15. Solucion 1 Los dgitos de las unidades colaboraran con 0 + 2 + 4 + 6 + 8 = 20 diez
veces. As mismo, los dgitos de las decenas tambien colaboraran con lo mismo otras
diez veces. De modo que la suma es 20 20 = 400.
Solucion 2 Podemos descartar al 100, pues no afecta la suma. Si escribimos a cada
n umero como un n umero de 2 dgitos (por ejemplo, al 7 como 07), no afectamos
la suma pedida, pero obtenemos la ventaja de que de los 2 100 = 200 dgitos que
se escriben en total, cada dgito de 0 a 9 aparece la misma cantidad de veces. En
particular, el 0, 2, 4, 6 y 8 aparecen cada uno 20 veces, y por tanto la suma pedida
es 20(0 + 2 + 4 + 6 + 8) = 20 20 = 400.
3.2. SEGUNDA ETAPA 33
16. Para eliminar los n umeros despues del dgito decimal, necesitamos en primer lugar
que c = 0. Fijando c = 0, tenemos ahora que b debe de ser 0, pero ya que deter-
minamos esto, a puede ser cualquier dgito. Esto quiere decir que hay 10 valores
posibles para k. Si tomamos a = 1 y a = 2, vemos que a + b + c no es constante,
pues en un caso vale 1 y en otro caso vale 2, y mas todava, vemos que no necesa-
riamente a = b = c. Finalmente, para vericar que 37 divide a k, observemos que
k = 100a + 10a +a = 111a = 37 3a.
17. Denotemos las longitudes de los arcos RS y PQ por x y y respectivamente. Por un
resultado conocido,
x+y
2
= PTQ = 20

, de modo que x + y = 40

. Esto es
1
9
del
permetro de 2 5 = 10 que tiene la circunferencia, de modo que la suma buscada
es
10
9
.
18. En la la j se escriben los n umeros de q(j 1) + 1 a qj. Como 20 esta en la tercer
la, 2q +1 20 3q. Como 41 esta en la quinta la, 4q +1 41 5q. Finalmente,
como 103 esta en la ultima (la la p), tenemos (p 1)q + 1 103 pq. La primer
desigualdad nos dice que q es 7, 8 o 9. La segunda nos dice que q es 8, 9 o 10. Esto
se resume a que q es 8 o 9.
Si q es 8, entonces 8(p 1) + 1 103 8p, de modo que p
102
8
+ 1 13,7, y
as p 13. Ademas, p
103
8
= 12,8, por lo que p 13. En este caso obtenemos
p = 13.
Si q es 9, entonces 9(p 1) + 1 103 9p, obtenemos de manera similar p = 12.
En cualquier caso, tenemos que p +q = 21.
19. La probabilidad de un evento es el n umero de casos favorables entre el n umero de
casos totales. En total tenemos 6 6 6 = 216 formas en las que pueden salir los dados.
Para contar los casos favorables, jemonos en el n umero que salio en el dado rojo. Si
es 6, entonces los dados negros tienen 5 5 opciones favorables: que en el primer dado
salgan uno de los cinco n umeros 1, 2, 3, 4 o 5 y que en el otro tambien. Si en el dado
rojo sale 5, entonces los dados negros tienen 4 4 posibilidades. Continuando de esta
forma, obtenemos un total de 5
2
+ 4
2
+ 3
2
+ 2
2
+ 1
2
=
5611
6
= 55 casos favorables.
Esto nos dice que la probabilidad buscada es
55
216
.
20. Llamemos = ACB y pongamos todo en terminos de . Como AC es perpendi-
cular a BD, tenemos que DBC = 90

. Como AB = AC, tenemos ABD =


ABC DBC = (90

) = 2 90

. Ahora, como BAD es isosceles y


el angulo desigual vale 2 90

, cada uno de los otros angulos vale 135

. As,
BCA+BDA = + 135

= 135

.
21. La edicion n umero d de la Olimpiada se celebra en el a no 1986 +d. Queremos que d
divida a 1986+d, pero como d divide a d, necesitamos que d divida a 1986. Hay tantos
a nos superolmpicos como divisores positivos de de 1986. Como 1986 = 2 3 331 es
la descomposicion en primos de 1986, tenemos que tiene 8 divisores y por tanto hay
8 a nos superolmpicos.
34 CAP

ITULO 3. SOLUCIONES DE LOS PROBLEMAS


22. Denotemos por d a la distancia de la Tierra a la Luna. Denotemos por a, b y c
las velocidades que cooperan los propulsores A, B y C respectivamente. Usando
que velocidad es distancia sobre tiempo, el problema nos da la informacion de que
a =
L
200
, a +b =
L
150
y que a +c =
L
120
. De este modo, al usar los propulsores B y C
tenemos una velocidad de b +c = (a+b) +(a+c) a =
L
150
+
L
120

L
200
=
(4+53)L
600
=
6L
600
=
L
100
. De este modo, para recorrer la distancia L necesitamos 100 horas usando
los propulsores B y C.
23. Los n umeros que tienen a lo mas un dgito 6 los podemos dividir en los siguientes
tres tipos ajenos:
Los que no tienen dgitos 6. En este caso, para el primer dgito hay 8 opciones
(no puede ser 0 ni 6) y para cada uno de los otros 3 hay 9 opciones. Esto nos
da un total de 8 9
3
= 5832 n umeros posibles.
Los que empiezan en 6 pero ya no tienen ning un otro 6. En este caso el primer
dgito ya esta determinado, y cada uno de los otros tiene 9 opciones, obteniendo
as 9
3
= 729 posibilidades.
Los que tienen solo un dgito 6 que no es el primero. En este caso hay que elegir
el primer dgito entre 8 posibilidades, decidir cual de los 3 restantes sera el
dgito 6, y los otros 2 elegirlos entre 9 posibilidades. Esto nos da un total de
3 8 9
2
= 1944 n umeros.
Sumando los tres casos, obtenemos un total de 8505 n umeros que satisfacen la con-
dicion deseada.
24. Sean A, B y C vertices consecutivos del hexagono. Sean ademas L el punto medio de
AB, M el punto medio de BC, O el centro del hexagono y P el centro del triangulo
OLM. Tenemos que OL = OM y que LOM = 60

, de modo que por criterio LAL


el triangulo LOM es equilatero. Esto nos dice quePLM = 30

, y ya tenamos que
BLMBAC = 30

. De este modo, los triangulos PLM y BLM comparten sus


angulos y su lado correspondiente LM, de modo que son congruentes. Esto nos dice
que el area del triangulo BLM es igual a la del PLM, que a su vez es un tercio
del equilatero LOM. As,
(LBM)
(LBMO)
=
1
4
, y repitiendo el argumento para los otros seis
triangulos amarillos, vemos que la razon total es
1
4
.
25. Solucion 1 Usaremos parte de la solucion de Primera Etapa Problema 20. De acuer-
do a ese problema, n! termina en tantos ceros como la cantidad de veces que 5
puede dividir a n. En nuestro caso, 24! tiene 4 factores 5, de modo que termina en
exactamente 4 ceros. Esto nos dice que c = 0.
Claramente 24! es divisible entre 9 y entre 11. El criterio de divisibilidad del 11 nos
dice que 11 divide a la suma alternada, que es b a 8. Como b a es un n umero
entre 9 y 9, la unica posibilidad para que haga un m ultiplo de 11 es si vale 3.
El criterio de divisibilidad del 9 nos dice que 9 divide a la suma de los dgitos, que
es 72 + a + b, es decir, queremos que a + b sea m ultiplo de 9, as que puede ser 0,
3.2. SEGUNDA ETAPA 35
9 o 18. Si es 0 o 18, ambos dgitos son iguales (a 0 o a 9), por lo que su resta es 0
y no cumple con la otra cosa que queremos. As, tenemos a +b = 9. Usando ambas
ecuaciones, obtenemos 2b = 6, por lo que b = 3 y a = 6.
Solucion 2 Del mismo modo obtenemos c = 0 y que a + b debe ser m ultiplo de 9.
Veamos que 24! es producto de al menos 12 n umeros pares y por tanto tiene a 2
como factor al menos 12 veces. As, tras dividirlo entre 10000 sigue siendo divisible
entre 8. De modo que por el criterio de divisibilidad del 8 el n umero de tres dgitos
9ab debe ser divisible entre 8.
Esto nos restringe a las siguientes posibilidades para el n umero de dos dgitos ab:
04, 12, 20, 28, 36, 44, 52, 60, 68, 76, 84 o 92. De estas, la unica con suma de dgitos
m ultiplo de 9 es 36. As, a = 3, b = 6.
36 CAP

ITULO 3. SOLUCIONES DE LOS PROBLEMAS


3.3. Tercera Etapa
Problema 1 Sea ABC un triangulo con AD la altura sobre BC. La circunferencia
de centro D y radio AD corta a AB en P y a AC en Q. Demuestra que los triangulos
ABC y AQP son semejantes.
Primer Solucion Sea E el punto de interseccion de AD con el crculo trazado. Como
AE pasa por el centro de la circunferencia, tenemos que es un diametro. As, EQC = 90

.
Tambien tenemos EDC = 90

. De modo que el cuadrilatero EDQC es cclico por abrir


el mismo angulo al lado CE. Usando este cuadrilatero cclico y el cuadrilatero cclico
APEQ obtenemos que ACB = AEQ = APQ. Ademas, los triangulos ABC y AQP
comparten el angulo en A, por lo que el criterio AA de semejanza nos garantiza que son
semejantes.
Segunda Solucion Consideremos la circunferencia de diametro AD y sus puntos de
interseccion P

con AB y Q

con BC. Como AD es diametro de la circunferencia, tenemos


que el angulo AQ

D es de 90

. As mismo, por ser AD altura, tenemos que ADC = 90

.
Juntando esta informacion con el cclico AP

DQ

tenemos que AP

= ADQ

=
90

DAQ

= ACB. Analogamente, podemos concluir que AQ

= ABC. Por el
criterio AA, conclumos que ABC y AQ

son semejantes.
Aplicando una homotecia de razon 2 en el vertice A, el crculo de diametro AD se
transforma en el crculo con centro en D y radio AD, de modo que P

se transforma en P
y Q

en Q. Esto nos dice que AQP y AQ

son homoteticos, y por tanto semejantes.


As, los triangulos AQP y ABC son semejantes.
Tercer Solucion Sean P

y Q

los pies de las alturas en el triangulo ABC desde


C y desde B y H el ortocentro de ABC. Es un hecho conocido que AQ

es semejante
3.3. TERCERA ETAPA 37
al triangulo ABC. Ademas, como el cuadrilatero AP

HQ

es cclico con centro en AH,


existe una homotecia que manda a la circunferencia del problema y por tanto P

a P y
Q

a Q. De este modo, AQP es semejante a AQ

y por tanto a ABC.


Problema 2 Fer, Gogo, Leo y Marco compran una barra de chocolate rectangular
de mn cuadritos unitarios. Van a jugar a lo siguiente sobre una mesa, alternandose de
la forma F, G, L, M, F, G, . . ., etc.
En el primer turno Fer toma el chocolate y lo corta en lnea recta por una de las
lneas que dividen a los cuadritos. En los turnos siguientes, de las piezas de chocolate en la
mesa el jugador correspondiente toma una de ellas (que no sea un cuadrito individual) y
hace lo mismo: cortarla por una lnea recta que pase por las divisiones de cuadritos. Gana
el jugador que al nalizar su turno deje la mesa con los mn cuadritos unitarios. Dados los
lados m y n del chocolate, determina que jugador puede ganar.
Solucion Notemos que en cada turno la cantidad total de piezas de chocolate en la
mesa aumenta en 1. El juego se termina cuando haya mn piezas de chocolate en la mesa,
e inicialmente hay una. De modo que sin importar como jueguen, el juego terminara en
mn1 pasos. De este modo, basta ver que residuo tiene mn1 al dividirse entre 4. Seg un
38 CAP

ITULO 3. SOLUCIONES DE LOS PROBLEMAS


el residuo sea 1, 2, 3 o 0 ganara respectivamente Fer, Gogo, Leo o Marco.
Problema 3 Sea n un entero mayor o igual a 10. A n se le quita el dgito de las
unidades y se obtiene un n umero m. Encuentra todos los n tales que m divide a n.
Solucion Si denotamos por k al dgito de las unidades de n, entonces tenemos
n = 10m + k con m 1 (pues n 10). Como queremos que m divida a n y m divide a
m, necesitamos que m divida a k. Tenemos dos casos:
Si k = 0 entonces cualquier m divide a k, y por tanto todos los enteros que terminan
en 0 mayores o iguales a 10 cumplen.
Si k es un dgito distinto de 0, entonces m tiene que ser un divisor de k y por tanto
tambien es un dgito. Los casos que tenemos son 11, 12, 13, . . ., 19, 22, 24, 26, 28,
33, 36, 39, 44, 48, 55, 66, 77, 88 y 99.

Problema 4 Pichi escribe todos los n umeros mayores a 10000 que se pueden formar
con dgitos a, b, c, d y e (no necesariamente en ese orden) que cumplen la condicion de
que b = a + 2 , c = b + 2, d = c + 2 y e = d + 2.
Cuantos n umeros escribio Pichi?
Calcula la suma de los n umeros que escribio Pichi.
Solucion Si a es 0 entonces el resto de los dgitos son 2, 4, 6 y 8. Si a es 1 entonces el
resto de los dgitos son 3, 5, 7 y 9. Estos son los unicos casos que tenemos. Consideremoslos
por separado:
Cuando los dgitos son pares, el primer dgito lo podemos elegir de 4 formas (todos
excepto el 0), el segundo de 4 formas, pero los siguientes de 3, 2 y 1 formas, pues
cada vez vamos agotando los dgitos. Esto nos da un total de 4 4! = 96 n umeros.
Para encontrar la suma veremos que sucede en cada uno de los dgitos. Notemos que,
por ejemplo, el dgito 2 aparece 4! = 24 veces como primer dgito, de modo que en esta
posicion colabora con 24 210000 a la suma total. En cualquier otra posicion aparece
3 3! = 18 veces, de modo que colabora con 18 2 (1+10+100+1000) = 18 2 (1111)
a la suma total. Para los otros dgitos tenemos una cuenta similar, de modo que
considerando todos los dgitos tenemos que la suma buscada es:
24 (2 + 4 + 6 + 8) 10
4
+ 18 (2 + 4 + 6 + 8) 1111 = 24 20 10
4
+ 18 20 1111
= 5199900
3.4. CUARTA ETAPA 39
Cuando los dgitos son impares, el primer dgito lo podemos elegir de 5 formas, y el
resto de 4, 3, 2 y 1 formas. As, en este caso tenemos 5! = 120 n umeros.
En este caso, cada dgito aparece en cada posicion el mismo n umero de veces, a saber
4! = 24. As, por ejemplo, el dgito 7 colabora con 247(1+10+100+1000+10000) =
24 7 11111 para la suma total, de modo que la suma esta dada por:
24 (1 + 3 + 5 + 7 + 9) 11111 = 24 25 11111
= 6666600
Recuperando la informacion de ambos casos, tenemos que Pichi escribio 96 +120 =
216 n umeros y su suma fue 5199900 + 6666600 = 11866500.
3.4. Cuarta Etapa
Primer Da
Problema 1 Un n umero natural de n dgitos es armonioso si sus n dgitos son
una permutacion de 1, 2, 3, . . . , n y sus primeros k dgitos forman un n umero divisible por
k, para k = 1, 2, 3, . . . , n. Por ejemplo, 321 es armonioso pues 3 es divisible por 1, 32 es
divisible por 2 y 321 es divisible por 3. Encuentra todos los n umeros armoniosos de 6
dgitos.
Solucion Supongamos que el n umero de 6 dgitos abcdef es armonioso. Descubri-
remos los dgitos mediante los siguientes pasos:
Como abcde es m ultiplo de 5, necesitamos que termine en 0 o 5, pero solo tenemos
los dgitos 1, 2, 3, 4, 5. As, e = 5.
Como ab, abcd, abcd5f son m ultiplos de 2, 4 y 6 respectivamente, entonces son
n umeros pares y por tanto terminan en un n umero par. Esto nos dice que b, d, f
son una permutacion de 2, 4, 6, de modo que a, c son una permutacion de 1, 3, y en
cualquier caso a +c = 4.
Necesitamos tambien que abc sea un m ultiplo de 3, es decir, que su suma de dgitos
sea m ultiplo de 3, pero su suma de dgitos es a +b +c = b +4. El unico b entre 2, 4
y 6 que hace esto es 4, as que b = 4.
Sabemos que abcd es divisible entre 4, de modo que por el criterio de divisibilidad del
4, necesitamos que cd sea divisible entre 4. Los valores que quedan para el n umero
de dos dgitos cd son 14, 16, 36 y 36 y solo 16 y 36 son m ultiplos de 4.
40 CAP

ITULO 3. SOLUCIONES DE LOS PROBLEMAS


Esto nos deja con dos casos, abcdef = 321654 y abcdef = 123654. El primer dgito
siempre es divisible entre 1, as que solo nos basta vericar que cada uno de estos
n umeros sea divisible entre 6. Pero esto es cierto, pues son pares y su suma de dgitos
es 21, un m ultiplo de 3.

Problema 2 Sea ABC un triangulo y sean D y E puntos sobre los segmentos AB


y AC tales que BD+CE = BC. Si I es el incentro del triangulo ABC, demuestra que el
cuadrilatero ADIE es cclico.
Solucion Como BD + BE = BC, podemos encontrar un punto F sobre BC tal
que BF = BD y CF = CE. Los triangulos BDI y BFI comparten su angulo en B por
ser I incentro, comparten el lado BI y ademas BD = BF. As, por criterio LAL son
congruentes. Analogamente, los triangulos CEI y CFI son congruentes.
De este modo, tenemos que ADI = 180

BDI = 180

BFI = IFC =
IEC = 180

AEI De este modo, los angulos en D y en E en el cuadrilatero ADIE


suman 180

y por tanto el cuadrilatero es cclico.


Problema 3 Demuestra que:
a)
1
3

1
2
+
3

12+
3

2
2
+
1
3

3
2
+
3

34+
3

4
2
+. . . +
1
3

999
2
+
3

9991000+
3

1000
2
>
9
2
b)
1
3

2
2
+
3

23+
3

3
2
+
1
3

4
2
+
3

45+
3

5
2
+. . . +
1
3

998
2
+
3

998999+
3

999
2
<
9
2
Solucion Sean
A =
1
3

1
2
+
3

1 2 +
3

2
2
+
1
3

3
2
+
3

3 4 +
3

4
2
+. . . +
1
3

999
2
+
3

999 1000 +
3

1000
2
B =
1
3

2
2
+
3

2 3 +
3

3
2
+
1
3

4
2
+
3

4 5 +
3

5
2
+. . . +
1
3

998
2
+
3

998 999 +
3

999
2
3.4. CUARTA ETAPA 41
Notemos primero que A > B, pues comparando termino a termino de izquierda a
derecha vemos que cada sumando de A es mayor a cada sumando de B y ademas A tiene
un sumando mas.
Cada uno de los sumandos de A o B es de la forma
1
3

j
2
+
3

j(j+1)+
3

(j+1)
2
. Usando
la formula a
3
b
3
= (ab)(a
2
+ab +b
2
), obtenemos
1
3

j
2
+
3

j(j+1)+
3

(j+1)
2
=
3

j+1
3

j
j+1j
=
3

j + 1
3

j. As, A + B es una suma telescopica en la cual los unicos sumandos que


quedan son
3

1000
3

1 = 10 1 = 9.
De forma que A+B = 9 y A > B, con lo cual necesariamente A >
9
2
y B <
9
2
.
Segundo Da
Problema 4 La FIFA desea cambiar la modalidad del mundial. En el torneo par-
ticiparan 32 equipos, los juegos de cada ronda se decidiran por sorteo y en cada partido
entre dos equipos exactamente uno, el ganador (no hay empates), pasara a la siguiente
ronda. La FIFA tiene un ranking de los 32 equipos ordenandolos de mejor a peor. En la
primera ronda del torneo se realizaran 16 partidos y los 16 ganadores pasan a la segunda
ronda, en la segunda ronda se jugaran 8 partidos y los 8 equipos ganadores pasan a la
tercera ronda as hasta que en la cuarta ronda habra 2 partidos y los ganadores jugaran
la nal. Supongamos que si un equipo A esta en mejor posicion en el ranking de la FIFA
que un equipo B entonces si A y B juegan, A le gana a B, por ejemplo el equipo 1 en el
ranking siempre gana. Bajo esta suposicion, cual es el peor equipo que puede disputar la
nal?
Solucion Numeremos los equipos de 1 a 32 de mejor a peor. Uno de los nalistas
viene de una llave en la cual jugaron 16 equipos y el otro seminalista de la otra, con
los 16 equipos restantes. As, para que un equipo llegue a la nal, tiene que ser el mejor de
su llave, y por tanto debe de ser capaz de ganarle a al menos 15 equipos. El peor equipo
que puede hacer eso es el equipo 17. Veamos que en efecto hay una conguracion que le
permita ganar.
Notemos que el equipo 1 siempre es seminalista, pues de hecho gana el mundial. Si
en la llave del equipo 1 juegan los equipos de 1 a 16, entonces el mejor equipo en la otra
llave es el 17 y por tanto gana todos sus partidos y llega a la nal. Esto nos muestra que
el equipo 17 puede llegar a la nal.
Problema 5 Sea ABC un triangulo con AB = AC y A = 36

. La bisectriz del
ABC corta a AC en D y la bisectriz del BDC corta BC en P. Sea R un punto sobre
la prolongacion de BC tal que B es el punto medio de PR. Demuestra que RD = AP.
Solucion Como A = 36

y ABC es isosceles, tenemos que B =


180

36

2
= 72

y as DBA = DBC = 36

. Tambien tenemos BDC = DBA + DAB = 72

, de
42 CAP

ITULO 3. SOLUCIONES DE LOS PROBLEMAS


modo que BDP = CDP = 36

.
Estos angulos nos dicen que BP = PD, que BD = DA y que PDA = 180

36

=
RBD. As, por criterio LAL los triangulos RBD y PDA son congruentes y por tanto
RD = AP.
Problema 6 Halla todos los enteros n > 1, tales que cada divisor primo de n
6
1
es divisor de n
2
1 o de n
3
1.
Solucion Tomemos un n > 1 que cumpla la condicion del problema. Tenemos la
factorizacion n
6
1 = (n + 1)(n 1)(n
2
+ n + 1)(n
2
n + 1). Como n
2
n + 1 =
n(n1) +1 > 2, entonces n
2
n+1 tiene un divisor primo p. Como p[n
2
n+1, tenemos
que p[n
6
1. De modo que por la condicion del problema, p[n
2
1 = (n + 1)(n 1)
o p[n
3
1 = (n 1)(n
2
+n + 1). Si p es primo y p[ab, entonces p[a o p[b, as que tenemos
tres casos:
Caso 1 p divide a n1. De ser as, p[(n
2
n+1) +(n1) = n
2
. Como p es primo,
tendramos p[n. As, p[(n
2
n + 1) n(n) + n = 1, lo cual no es posible. En este
caso no hay n que cumpla.
Caso 2 p divide a n
2
+n+1. A partir de eso, conclumos que p[(n
2
+n+1) (n
2

n+1) = 2n. Si p[n, entonces llegamos a algo imposible como en el inciso anterior, de
modo que p[2. Como p era cualquier divisor de n
2
+n+1, tenemos que n
2
+n+1 = 2
k
para alg un entero positivo k.
Si k 2, entonces el lado derecho es divisible entre 4, pero modulo 4 los unicos valores
posibles de n
2
n+1 son 1 y 3. As, de existir solucion debe ser con k = 1, de donde
n
2
n + 1 = 2. Pero esto es imposible pues ya habamos dicho que n
2
+n + 1 > 2.
De nuevo no tenemos soluciones.
Caso 3 p divide a n +1. Entonces p tambien divide a (n +1)(n 2) = n
2
n 2 y
3.5. QUINTA ETAPA 43
por tanto p[(n
2
n +1) (n
2
n 2) = 3. De modo que p = 3 y entonces tenemos
que n
2
n + 1 = 3
k
para alg un entero positivo k.
Si k 2, entonces el lado derecho es divisible entre 9, pero modulo 9 los unicos
valores posibles de n
2
n + 1 son 1, 2, 4, 5, 7 y 8. As, de existir una solucion,
tendra que cumplir n
2
n + 1 = 3, o bien n(n 1) = 2. Vemos por inspeccion que
n = 2 es la unica solucion positiva de esta ecuacion (la otra solucion es n = 1).
Esto nos forza a que si existe una solucion, entonces debe ser n = 2. Todava queda
por ver que en efecto n = 2 es solucion. Tenemos que n
6
1 = 63. Los unicos divisores
primos de 63 son 3 y 7. Y en efecto, tenemos que 3 divide a 2
2
1 = 3 y que 7 divide a
2
3
1 = 7. Conclumos que n = 2 es la unica solucion.
3.5. Quinta Etapa
Primer Examen, Primer Da
Problema 1 Se tiene el n umero 1234 escrito en una hoja blanca. Cesar y Lalo
toman turnos para jugar lo siguiente: en cada turno un jugador puede restarle cualquiera
de los dgitos del n umero escrito en la hoja (siempre y cuando este dgito no sea cero);
se borra el viejo n umero y se escribe el nuevo. Pierde el que ya no pueda restar n umeros
(cuando se llegue al n umero cero). Si Cesar es el que empieza, quien gana y cual es su
estrategia ganadora?
Solucion Cesar gana. Para derrotar a Lalo debe restar siempre el ultimo dgito.
Con esto, logra que el n umero que quede termine en 0. Como Lalo debe restar un dgito
que no sea 0, entonces cambia el valor del ultimo dgito. As, si Cesar juega as, puede
garantizar que Lalo siempre dejara el ultimo dgito distinto a cero, por lo que Cesar
siempre tendra jugada. Como los n umeros que se obtienen son estrictamente decrecientes,
llegara un momento en el que el n umero en la hoja sea 0, y ya demostramos que no puede
ser el turno de Cesar, de modo que es Lalo quien ya no puede hacer movimientos.
Problema 2 Sea ABC un triangulo rectangulo con angulo recto en B. Sea E el pie
de la altura desde B y consideremos I
A
e I
C
los incentros de ABE y CBE respectivamente.
Demuestra que AI
A
es perpendicular a BI
C
.
Demuestra que la bisectriz por B es perpendicular a I
A
I
C
.
Solucion Sea P la interseccion de AI
A
con BI
C
. Notemos que BAE = 90


BCE = EBC. Analogamente, ABE = BCE. As, ABP + BAP = ABE +
44 CAP

ITULO 3. SOLUCIONES DE LOS PROBLEMAS


EBC
2
+
BAE
2
= ABE+BAE = 90

. Esto nos dice que APB = 90

, como queramos.
Esto demuestra la primer parte.
De modo similar, podemos demostrar que CI
C
es perpendicular a BI
A
. Denotemos
el punto de interseccion de AI
A
y CI
C
como I. Como esta en dos bisectrices, es el incentro
de ABC. As, la tercer bisectriz tambien pasa por I. Pero como esta en dos alturas de
BI
A
I
C
, entonces es el ortocentro de este triangulo, de modo que la recta BI, la bisectriz
en B del triangulo ABC, es perpendicular a I
A
I
C
.
Problema 3 Sean a, b y c n umeros reales mayores que cero. Demostrar que:
a
b
+
b
c
+
c
a

_
b
a
_
3
+
_
c
b
_
3
+
_
a
c
_
3
Solucion Multiplicando ambos lados de la desigualdad por (abc)
3
, queremos de-
mostrar que a
4
b
2
c
3
+ b
4
c
2
a
3
+ c
4
a
2
b
3
a
6
b
3
+ b
6
c
3
+ b
6
c
3
. Aplicando MA-MG para 9
n umeros, tenemos que:
b
6
c
3
+ 4c
6
a
3
+ 4a
6
b
3
9

9

a
36
b
18
c
12
= a
4
b
2
c
3
Encontrando desigualdades similares rotando a, b y c y sumandolas, obtenemos la
desigualdad buscada.
Primer Examen, Segundo Da
Problema 4 Determinar todos los n umeros primos p tales que 5
p
+ 4p
4
es un
cuadrado perfecto.
Solucion Haciendo 5
p
+4p
4
= n
2
obtenemos que 5
p
= n
2
4p
4
= (n+2p
2
)(n2p
2
).
Si ambos terminos a la derecha son m ultiplos de 5, entonces en particular su diferencia,
3.5. QUINTA ETAPA 45
4p
2
, tambien es m ultiplo de 5 y por tanto p = 5. En efecto, esto es solucion, pues 5
5
+45
4
=
5
4
(5 + 4) = 3
2
5
2
, que es un cuadrado perfecto.
En otro caso, tenemos que uno de los factores n +2p
2
y n 2p
2
es 1 y el otro es 5
p
.
Como p es primo, es mayor o igual a 2 y por tanto n + 2p
2
no puede ser 1. As, tenemos
que n 2p
2
= 1 y n + 2p
2
= 5
p
. Restando ambas ecuaciones obtenemos que 4p
2
= 5
p
1.
Esto no es posible pues 5
p
1 = 4(5
p1
+ 5
p2
+ . . . + 5 + 1) > 4 5
p
> 4p
2
(donde la
ultima desigualdad se puede demostrar inductivamente para los n umeros enteros n 2).
De esta forma, tenemos que la unica solucion es p = 5.
Problema 5 Sea ABC un triangulo y G su gravicentro. Se construyen triangulos
equilateros exteriores BCD, CAE y ABF con gravicentros A

, B

y C

. Demuestra que
G es el circuncentro del A

.
Solucion Para ver que es el circuncentro, demostraremos que GA

= GB

= GC

.
Notemos que los triangulos FAC y BAE son congruentes por criterio LAL, pues AB =
AF, AC = AE y FAC = BAE. Esto nos dice que FC = BE.
Tracemos los puntos medios L y M de AF y AC respectivamente. Por la razon
que hace el gravicentro en la mediana, tenemos que
BB

BL
=
2
3
=
BG
BM
. As, los triangulos
BB

G y BLM son semejantes en razon


2
3
. Como ademas
AL
AF
=
1
2
=
AM
AC
, los triangulos
ALM y AFC son semejantes en razon
1
2
. De este modo, tenemos que GB

=
2
3
LM =
FC
2
.
46 CAP

ITULO 3. SOLUCIONES DE LOS PROBLEMAS


Analogamente obtenemos GC

=
BE
2
. Pero habamos probado al inicio que FC = BE, a
partir de lo cual conclumos que GB

= GC

. Simetricamente obtenemos GB

= GA

y
as G es circuncentro de A

.
Problema 6 Lalo y Cesar juegan volados. Lanzan n veces la moneda. Cesar gana
si la cantidad de aguilas obtenidas es m ultiplo de 4, y Lalo gana en otro caso. Para que n
la probabilidad de que gane Cesar es
1
4
?
Solucion Para encontrar la probabilidad de un evento, necesitamos dividir el n ume-
ro de casos favorables entre el n umero de casos totales. En total hay 2
n
formas en las cuales
puede salir el resultado de tirar n monedas. Para que Cesar gane, es necesario que salgan
0, 4, 8, . . ., monedas, y tirando n monedas esto se puede hacer de
_
n
0
_
,
_
n
4
_
,
_
n
8
_
, . . . formas
respectivamente. Entonces lo que queremos encontrar son las n para las cuales:
X =
_
n
0
_
+
_
n
4
_
+
_
n
8
_
+. . .
2
n
=
1
4
Consideremos el n umero complejo i, es decir, un n umero tal que i
2
= 1. Usando el
Binomio de Newton:
A = (1 + 1)
n
=
_
n
0
_
+
_
n
1
_
+
_
n
2
_
+
_
n
3
_
+
_
n
4
_
. . .
B = (1 1)
n
=
_
n
0
_

_
n
1
_
+
_
n
2
_

_
n
3
_
+
_
n
4
_
. . .
C = (1 +i)
n
=
_
n
0
_
+
_
n
1
_
i
_
n
2
_

_
n
3
_
i +
_
n
4
_
. . .
D = (1 i)
n
=
_
n
0
_

_
n
1
_
i
_
n
2
_
+
_
n
3
_
i +
_
n
4
_
. . .
Notemos que:
X =
A+B +C +D
4 2
n
=
2
n
+ (1 +i)
n
+ (1 i)
n
2 2
n
=
1
4
+
(1 +i)
n
+ (1 i)
n
2 2
n
As, tras igualar esta expresion a
1
4
, vemos que buscamos las n para las cuales
(1 +i)
n
+ (1 i)
n
= 0. Como la norma (1 +i)
n
y de (1 +i)
n
es la misma, es necesario y
suciente que queden alineados con el origen y de lados opuestos con respecto a el para
3.5. QUINTA ETAPA 47
que su suma sea 0. El angulo con el eje x de (1+i) al elevarlo en potencias va aumentando
en

4
y el de (1 +i) va disminuyendo en

4
. As, quedan alineados como queremos si y solo
si n deja residuo 2 al dividirse entre 4.
Segundo Examen, Primer Da
Problema 1 Encuentra todos los enteros positivos n tales que 3n 4, 4n 5 y
5n 3 sean simultaneamente primos.
Solucion Notemos que los n umeros 3n 4 y 5n 3 tienen suma impar 8n 7,
por lo que son de distinta paridad. Esto nos dice que alguno de ellos es 2. Si 3n 4 = 2,
entonces n = 2, y obtenemos una solucion pues 4n 5 = 3 y 5n 3 = 7. El otro caso es
que 5n 3 = 2, de donde obtenemos n = 1. Pero entonces 3n 4 = 1 y por tanto n = 1
no es solucion.
Problema 2 Sea a
1
, a
2
, a
3
, . . ., una sucesion de n umeros de tal manera que a
1
= 1
y para n > 1, a
1
+a
2
+. . . +a
n
= n
2
a
n
. Encuentra a
2010
.
Solucion De la condicion del problema tenemos las siguientes dos ecuaciones:
a
1
+a
2
+. . . +a
n
= n
2
a
n
a
1
+a
2
+. . . +a
n
+a
n+1
= (n + 1)
2
a
n+1
Restando la primer ecuaci on de la segunda obtenemos que a
n+1
= (n + 1)
2
a
n+1

n
2
a
n
. Despejando a
n+1
obtenemos que:
a
n+1
=
n
n + 2
a
n
As, podemos encontrar a
2010
de la siguiente manera:
a
2010
=
2009
2011

2008
2010

2007
2009
. . .
2
4

1
3
a
1
=
2
2010 2009
De modo que el n umero buscado es
2
20102009
.
Problema 3 Sea M el punto medio del segmento AB. Dos circunferencias, una que
pasa por A y M, y otra que pasa por B y M, se intersectan en M y N. Sean P el punto
48 CAP

ITULO 3. SOLUCIONES DE LOS PROBLEMAS


medio del arco AN que no contiene a M en el circuncrculo del triangulo MAN y Q el
punto medio del arco BN que no contiene a M en el circuncrculo del triangulo MBN.
Demuestra que las rectas MN y PQ son perpendiculares entre s.
Solucion 1 Vamos a darle la vuelta al problema. Tomemos A, B, M y N como
en el problema, pero a un no consideremos a P y Q como los puntos medios. Mas bien,
sobre MN eligiremos el punto C tal que MC = MA = MB. Denotemos por a MAC
y por a MBC. Por la construccion del punto C, obtenemos que MCA = y que
MCB = . As, en el triangulo ABC tenemos 2( +) = 180

, de modo que el angulo


ACB es recto.
Nombremos K, L a las intersecciones de AC y BC respectivamente con el crcuncrcu-
lo de AMN. Nombremos P

y Q

como en la gura. Nuestro objetivo ahora se convierte


en mostrar que P = P

, que Q = Q

y que KL es perpendicular a MN.


Como C esta en el eje radical de las circunferencias, su potencia hacia ambas es la
misma, de modo que CK CA = CL CB. Esto muestra que el cuadrilatero ABKL es
cclico, y que por tanto BLK = . Pero por angulos opuestos por el vertice C, tenemos
que LCN = . Con esto conclumos que KL es perpendicular a MN.
Finalmente, veremos que P = P

y que Q = Q

. En el cclico AMKP

tenemos que
MP

K = . Esto nos dice que MP

es paralela a CL y por tanto perpendicular a AK.


Como el triangulo AMC es isosceles y MP

es altura, entonces MP

tambien es bisectriz
y por tanto P = P

. De manera an aloga, Q = Q

.
Solucion 2 Esta vez daremos una solucion directa, pero con una construccion si-
milar. En esta ocasion consideraremos el punto C sobre MN tal que MC = MA = MB
pero sobre el rayo NM. Usaremos la gura como referencia. Nombramos = MAC y
= NMQ = QMB. Los puntos K y L se construyeron como las intersecciones de AC
con la primer circunferencia y de AB con la segunda.
Como en la solucion anterior, tenemos por construccion que MCB = MBC y
3.5. QUINTA ETAPA 49
que + = 90

. Al igual que en la solucion anterior, por un argumento de potencia de


un punto podemos concluir que el cuadrilatero ALBK es cclico, de donde LKC = .
Esto nos muestra que CM es perpendicular a LK. Procederemos a probar que LK y PQ
son paralelas, con lo cual lograremos demostrar lo pedido.
Notemos que el angulo externo al triangulo MCB en M es 2, de modo que
MCB = MBC = . Esto nos dice que las rectas CK y MQ son paralelas.
Todos los trapecios cclicos son isosceles, y BKQM es un trapecio cclico. A partir
de esto obtenemos que MQK = y por lo tanto que CKQM es un paralelogramo.
Como en los paralelogramos tenemos lados opuestos iguales, deducimos que QK = CM.
De manera analoga podemos demostrar que LP = CM. As, el cuadrilatero LKQP tiene
dos lados iguales y paralelos y por tanto tambien es paralelogramo. En particular, LK y
PQ son paralelas, tal como queramos.
Nota Tras aplicar una inversion de radio arbitrario en el punto M se obtiene el
siguiente problema:
Problema Sea ABC un triangulo y M el punto medio de BC. La bisectriz de AMC
corta a AC en D y la bisectriz de AMB corta a AB en E. El circuncrculo de DME corta
a AM en P. Demuestra que PM es un diametro del circuncrculo de EMD.
Resolver este problema da otra solucion al problema original. Este es un problema
bastante mas sencillo y se deja como practica.
50 CAP

ITULO 3. SOLUCIONES DE LOS PROBLEMAS


Segundo Examen, Segundo Da
Problema 4 Un triangulo ABC es tal que su angulo en A es de 60

. Sean D y E
puntos sobre los lados AB y AC respectivamente de manera que BD = DE = EC. Sea O
el punto de interseccion de BE y DC. Demuestra que O es el circuncentro del triangulo
ABC.
Solucion
Como los triangulos DEC y BDE son isosceles, podemos nombrar los angulos como
en la gura.
Por angulos externos, tenemos DEA = 2 y EDA = 2. Por suma de angulos
en el triangulo ADE tenemos que 2 + 2 + 60

= 180

, de donde podemos deducir que


+ = 60

. De este modo, DOE = 180

= 120

. As, el cuadrilatero ADOE


tiene dos angulos opuestos que suman 180

y por tanto es cclico.


Entonces, OAE = = OCA. Esto nos dice que el triangulo OCA es isosceles,
de modo que OC = OA. Analogamente conclumos que OB = OA y que por tanto O es
el circuncentro del triangulo ABC.
Problema 5 Un marciano est a atrapado en un cuadrito de 11 de una cuadrcula de
nn donde n es un entero positivo impar mayor que 1. Se ha enviado una nave a rescatarlo,
pero la nave no logra ver al marciano. La nave puede teletransportar al marciano abordo
con disparos de rescate, cada uno de ellos cubre tres cuadros de la siguiente forma:
Los disparos se pueden encimar, rotar y salir de la cuadrcula. Cual es el mnimo
n umero de disparos de rescate que tiene que hacer la nave para asegurarse de haber
rescatado al marciano?
3.5. QUINTA ETAPA 51
Solucion Consideraremos el caso en el que n es m ultiplo de 3 y en el que no. En
cualquier caso, el tablero tiene
n
2
3
casillas en las cuales la nave debe buscar. Cada disparo
cubre tres casillas. Usaremos frecuentemente que con dos disparos se puede cubrir un
rectangulo de 2 3.
Si n no es m utiplo de 3, entonces
_
n
2
3
_
no bastaran, pues con estos no se alcanza a
cubrir en n umero la cantidad total de casillas. As, al menos se necesitan
_
n
2
3
_
+1 disparos.
Vamos a construir ejemplos que muestren que esta cantidad de disparos es suciente. A
continuacion mostramos acomodos para tableros de lado 5 y 7 que usan 9 y 17 disparos
respectivamente (dos por cada rectangulo de 2 3 y uno mas para el centro):
Cada uno de estos acomodos los podemos extender en 6 unidades por lado, pues con
rectangulos de 2 3 se puede construir cualquier tablero de 6 n para n 5. Esto nos
permite dar acomodos exactos para cualquier n umero de la forma 6k + 1 y 6k + 3.
Ahora, si n es m ultiplo de 3, en cuanto a cantidad es posible que
_
n
2
3
_
disparos
fueran sucientes, pero veremos que no es as. Si se pudiera cubrir entonces la forma de
hacer los disparos sera muy precisa y no debera haber dos disparos que se encimen.
Al pintar el tablero como tablero de ajedrez (digamos, con la esquina superior izquierda
negra), vemos que cada disparo cubre exactamente tres cuadros del mismo color. Como
los disparos no se enciman, entonces debe haber una cantidad m ultiplo de 3 de cuadros
negros y blancos. Pero como el tablero es de lado impar, hay exactamente un cuadro negro
mas que cuadros blancos, por lo que no pueden ambos n umeros ser m ultiplos de 3. Esta
es una contradiccion que nos dice que tal acomodo exacto no existe.
As, en este caso debemos usar tambien al menos
_
n
2
3
_
+ 1 disparos de rescate. Es
facil cubrir un tablero de 3 3 con 4 disparos. Para cualquier otro n umero de la forma
6k + 3, hacemos mas grande el acomodo como en el caso anterior.
Nota El problema se puede resolver para tableros que no necesariamente tienen
lado impar. El unico caso al cual le caben exactamente los disparos necesarios (
_
n
2
3
_
) es
el tablero de 6 6. Los otros casos necesitan
_
n
2
3
_
+ 1 disparos, lo cual se puede probar
con argumentos muy similares a los que ya se dieron.
Problema 6 Determina todas las parejas (a, b) de enteros positivos tales que ab
2
+9
52 CAP

ITULO 3. SOLUCIONES DE LOS PROBLEMAS


divide a a
2
b +b.
Solucion Como ab
2
+ 9 divide a a
2
b + b y a ab
2
+ 9, entonces divide a cualquier
combinacion lineal de estos dos n umeros. En particular, divide a b(a
2
b +b) a(ab
2
+9) =
b
2
9a. Tenemos tres posibilidades:
Caso 1 El n umero b
2
9a es positivo. Esto no es posible, pues ab
2
+ 9 tambien es
positivo y ab
2
+ 9 > b
2
> b
2
9a, pero un n umero positivo no puede dividir a un n umero
positivo mas chico.
Caso 2 El n umero b
2
9a es cero. En este caso, b
2
= 9a y entonces podemos escribir
a = k
2
con k un entero positivo, de donde b = 3k. Haciendo estas substituciones en la
condicion original, obtenemos que 3k
4
+9[3k
5
+3k. Ademas, 3k
4
+9[k(3k
4
+3) = 3k
5
+3k.
Restando ambas divisibilidades, obtenemos que 3k
4
+ 9[6k. Pero para cualquier valor de
k el lado izquierdo es mas grande, de modo que en este caso tampoco hay solucion.
Caso 3 El n umero b
2
9a es negativo. En este caso, ab
2
+9 divide al n umero positivo
9a b
2
. Si b es mayor o igual a 3, entonces ab
2
+9 9a +9 > 9a b
2
, y de nuevo no hay
solucion en este caso. As, solo quedamos con la posibilidad de que b sea igual a 1 o 2.
Si b es igual a 1, de ab
2
+9[9ab
2
obtenemos que a+9[9a1. Pero a+9[9(a+9) =
9a+81, de modo que restando ambas divisibilidades obtenemos que a+9[82. Los divisores
de 82 son 1, 2, 41 y 82. Como a es positivo, tenemos que a + 9 debe ser 41 o 82. Esto nos
da respectivamente a = 32 o a = 73. Se puede vericar que en efecto las parejas (32, 1) y
(73, 1) son solucion al problema.
Si b es igual a 1, de ab
2
+ 9[9a b
2
obtenemos que 4a + 9[9a 4, de donde 4a +
9[4(9a 4) = 36 16. Pero 4a + 9[9(4a + 9) = 36a + 81. Restando esta informacion,
obtenemos que 4a + 9[97. Como 4a + 9 > 1 y 97 es primo, debemos tener 4a + 9 = 97, de
donde 4a = 88 y por lo tanto a = 22. De nuevo, se puede vericar que en efecto (22, 4) es
solucion.
As, las unicas soluciones son (32, 1), (73, 1) y (22, 4).
3.6. Examen de Desempate
Problema 1 Sea ABCD un cudrlatero cclico con diagonales perpendiculares. Sea
E el punto de interseccion de la perpendicular a AB por B y la paralela a BD por C y
sea F el punto de interseccion de AE y BD. Demuestra que los triangulos AFD y ABC
son semejantes.
Solucion Como EC es paralela a BD y BD es perpendicular a AC, entonces EC es
perpendicular a AC. As, en el cuadrilatero BECA los angulos opuestos en B y C suman
180

y por tanto el cuadrilatero es cclico. Esto nos dice que E cae en la circunferencia del
problema. Como BECD es un trapecio cclico, entonces es isosceles y por tanto CAD =
DBC = EDB = BAE.
3.6. EXAMEN DE DESEMPATE 53
Usando los cinco puntos cclicos obtenemos que ADF = ACB. Ademas, FAD =
FAC + CAD = FAC + BAE = BAC. As, por el criterio de semejanza AA los
triangulos ABC y AFD son semejantes.
Problema 2 Sean a, b y p enteros no negativos con p un n umero primo. Encontrar
todas las ternas (a, b, p) tales que a
2
b
2
= 2
2010
pb.
Solucion Sumando pb de ambos lados y factorizando b obtenemos que b(a
2
b +p) =
2
2010
. Como el lado derecho es una potencia de 2, cada uno de los factores del lado izquierdo
tambien debe serlo. Dividiremos en casos seg un que potencia de 2 es b.
Si b es 1, entonces necesitamos que a
2
+ p = 2
2010
. Restando a
2
de ambos lados
obtenemos que p = 2
2010
a
2
= (2
1005
a)(2
1005
+ a). Como p es primo, uno de estos
factores debe ser 1 y el otro debe ser p. De hecho, el factor chico, 2
1005
a debe ser 1 y
2
1005
+ a debe ser p. Pero esto no es posible, pues entonces tendramos p = 2
1005
+ a =
2
1
005 + 2
1005
1 = 2
1006
1 = (2
503
1)(2
503
+ 1), factorizando al primo p. As, en este
caso no hay solucion.
Si b es 2, entonces la ecuacion se transforma en 2(2a
2
+p) = 2
2010
, o bien 2a
2
+p =
2
2009
. Si p es primo impar, el lado izquierdo es un impar mayor a 1, y esto es imposible
pues seg un el lado derecho no debe haber factores impares. As, p tiene que ser igual a 2 y
obtenemos 2a
2
+2 = 2
2009
, o bien a
2
+1 = 2
2008
. Pero tomando modulo 4 de ambos lados
vemos que esto es imposible pues no existe un cuadrado congruente con 1 modulo 4. De
este modo, para b = 2 tampoco tenemos soluciones.
Esto nos dice que si existe una solucion, entonces b es una potencia de 2 mayor a 2,
digamos 2
k
con k 2. De este modo, la ecuacion original se transforma en 2
k
(2
k
a
2
+p) =
2
2010
. Si p es impar, entonces de nuevo el lado izquierdo tiene un factor impar mayor a
1, lo cual es imposible. Si p es 2, entonces cancelando un 2 de ambos lados obtenemos
que 2
k
(2
k1
a
2
+ 1) = 2
2009
. Como 2
k1
a
2
+ 1 es un n umero impar (pues k 2) y al lado
izquierdo tenemos una potencia de 2, necesitamos forzosamente que este n umero sea 1 y
54 CAP

ITULO 3. SOLUCIONES DE LOS PROBLEMAS


por tanto que a sea 0. Esto nos dice que k = 2009.
Finalmente, vericamos que en efecto p = 2, a = 0 y b = 2
2009
es una solucion para
la ecuacion y por tanto es la unica solucion.
Problema 3 Toma k entero positivo jo. Los vertices de un nagono regular se
quieren pintar con k colores distintos de modo que cada que los vertices A
1
, A
2
y A
3
estan
pintados del mismo color y los vertices B
1
, B
2
y B
3
estan pintados del mismo color (quizas
distinto al de los vertices A
i
), entonces los triangulos A
1
A
2
A
3
y B
1
B
2
B
3
comparten al
menos un punto (en su interior, lados, o vertices). Cual es el mayor valor que puede tomar
n?
Solucion Demostraremos que el maximo valor que puede tomar n para k > 1 es 4k.
Para eso daremos una coloracion de un 4kagono regular que cumpla lo pedido y veremos
que con mas vertices no es posible dar tal coloracion. Al nal trataremos el caso k = 1.
Tomemos (1), (2), . . ., (k) los colores. Para dar una coloracion en el polgono de 4k
vertices, coloreamos de la siguiente manera (1)(1)(2)(2) . . . (k)(k)(1)(1)(2)(2) . . . (k)(k) en
el sentido de las manecillas del reloj. Al tomar 3 vertices de un mismo color, forzosamente
tomamos dos de ellos diametralmente opuestos, de modo que un lado del triangulo que
forman es un diametro del polgono. Como cualesquiera dos diametros tienen interseccion
no vaca (tienen al centro del polgono al menos), entonces cualesquiera dos triangulos
como en el problema se intersectan.
Tomemos ahora un polgono con n 4k + 2 lados con cualquier coloracion. Al
tomar un arco de 2k + 1 vertices consecutivos y su complemento, por principio de las
casillas en cada uno de ellos hay 3 vertices de un mismo color, pero como son arcos
complementarios, los triangulos que denen dichos vertices no se intersectan. Asi, en este
caso no hay coloracion posible.
Finalmente, descartaremos el caso n = 4k + 1. Llamemos a
j
al arco de 2k vertices
consecutivos en sentido horario que comienza en el vertice j (para 1 j n). Armamos
que uno de estos arcos tiene 3 vertices del mismo color. Si as no fuera, en cada arco
aparece a lo mas 2 veces cada color. Hay 4k +1 arcos y cada vertice esta en 2k arcos. As,
si contamos las apariciones del color c tenemos que son a lo mas
_
2(4k+1)
2k
_
=
_
4 +
1
k
_
= 4
(aqu es donde usamos k > 1). De modo que sumando sobre los colores, a lo mas hay 4k
vertices, pero esto es una contradiccion.
De esta manera, hay un arco de 2k vertices consecutivos con 3 vertices del mismo
color. Por principio de las casillas, en su complemento, que tiene 2k + 1 vertices, tambien
hay 3 vertices del mismo color. Estos vertices nos dan triangulos cada uno con vertices
del mismo color que no se intersectan, por lo que n = 4k + 1 tampoco se puede colorear.
Finalmente, resolveremos el caso k = 1. Si ponemos 6 vertices, entonces al tomar
tres de ellos consecutivos y al tomar los otros tres tenemos dos triangulos con vertices del
mismo color que no se intersectan. De modo que n es menor o igual a 5.
Pero claramente al tomar dos triangulos con 5 posibles vertices estos triangulos se
3.6. EXAMEN DE DESEMPATE 55
intersectan, as que n = 5 s cumple la condicion y por tanto en este caso es el maximo
valor que puede tomar n.
56 CAP

ITULO 3. SOLUCIONES DE LOS PROBLEMAS


Captulo 4
Examenes de Practica de Temas
Especcos
Esos fueron algunos de los examenes de practica de temas especcos que se resol-
vieron durante los entrenamientos.

Algebra
1. Se colocan los enteros positivos de 1 a n
2
en orden en un tablero de n n. Cual
es la suma de los elementos en la diagonal que va del cuadro superior izquierdo al
cuadro inferior derecho? Cual es la suma de los elementos en la otra diagonal?
2. Simplica la siguiente expresion
(2 + 1)(2
2
+ 1)(2
4
+ 1)(2
8
+ 1) (2
2
2010
+ 1)
3. Los reales positivos x, y y z tienen suma 2010. Demuestra que si
x +y z
x
2
+ 2yz
=
y +z x
y
2
+ 2zx
=
z +x y
z
2
+ 2xy
entonces cada una de estas fracciones es igual a
1
2010
.
4. Sea a un n umero real tal que a
1694
y a
585
son n umeros racionales. Demuestra que
a
2010
tambien es un n umero racional.
5. Los n umeros x, y y z son reales positivos tales que
1
x
+
1
y
+
1
z
=
1
x +y +z
Demuestra que:
57
58 CAP

ITULO 4. EX

AMENES DE PR

ACTICA DE TEMAS ESPEC

IFICOS
1
x
3
+
1
y
3
+
1
z
3
=
1
(x +y +z)
3
6. Demuestra la siguiente desigualdad para qualesquiera m y n enteros mayores o igua-
les a 1:
(m+n)
m+n
(m+n)!
>
m
m
m!

n
n
n!
Combinatoria
1. Se tienen los n umeros 1, 2, . . ., 10 en el pizarron. Si en un momento se tienen a
1
,
. . ., a
10
en el pizarron, entonces se pueden cambiar por:
a
1
+a
2
4
,
a
2
+a
3
4
, . . . ,
a
10
+a
1
4
o por:
2a
1
a
2
2
,
2a
2
a
3
2
, . . . ,
2a
10
a
1
2
Demuestra que sin importar que pasos se den, al sexto paso la suma de los n umeros
sera menor a 1.
2. Tomamos f(x) = (

x 3)
2
. Encuentra el valor de f
33
(10000)
1
.
3. En la danza nocturna gatuna participan 50 gatos: 10 de ellos blancos, 10 de ellos
cafes y 30 de ellos negros. Para bailar se dividen en 5 rondas de 10 gatos. Al llenarse
la luna, los gatos bailan y bailan y despues de esto vuelven a formar 5 rondas de 10
gatos, no necesariamente las mismas. Demuestra que hay dos gatos del mismo color
que estuvieron en la misma ronda antes y despues de la luna llena.
4. Dos jugadores, A y B, juegan a tachar los n umeros de 1 a 10, pero el jugador que
tacha un n umero debe tachar todos sus divisores que no esten tachados todava.
Gana el jugador que tache el ultimo n umero. A juega primero. Demuestra que A
tiene una estrategia ganadora.
5. Demuestra que no es posible cubrir un tablero de 9 8 con chas de 1 6.
6. Escribimos la secuencia formada por los n umeros naturales en orden:
S = 12345678910111213 . . .
1
Denimos f
1
(x) = f(x) y f
n
(x) = f(f
n1
(x)), es decir, f
n
(x) = f(f(f(. . . f(f(x)) . . .)) en donde esta
n veces la funci on f.
59
Para una n ja, denimos f(n) = m si el dgito 10
n
de izquierda a derecha en S
esta en los n umeros de m dgitos. Por ejemplo, f(0) = 1, pues el dgito 10
0
= 0 en
S esta en los n umeros de 1 dgito, f(1) = 2, pues el dgito 10 de S es el 1 de 10, que
esta en los numeros de 2 dgitos, y f(2) = 2, pues el dgito 100 de S es el 5 de 55,
que es un n umero de 2 dgitos. Encuentra con justicacion f(2010).
Geometra
1. Sea ABCD un cuadrilatero ciclico en el que se cumple AB+CD = BC. Demuestra
que las bisectrices de los angulos DBA y CDA se cortan sobre BC.
2. Dos crculos
1
y
2
se intersectan en los puntos M y N. La tangente com un a ambos
crculos mas cercana a M toca a
1
en A y a
2
en B. La paralela a AB por M
corta de nuevo a
1
y
2
en C y D respectivamente. AC y BD se intersectan en E.
CD intersecta a AN y BN en P y Q respectivamente. Demuestra que EP = EQ.
3. Sea ABC un triangulo acutangulo con AB < AC. Sea H su ortocentro y M el punto
medio de BC. Sea l la bisectriz de BHC. La paralela a l por M intersecta AC en
un punto N (N entre A y C). Demuestra que el permetro del NMC es igual al
permetro del cuadrilatero NMBA.
60 CAP

ITULO 4. EX

AMENES DE PR

ACTICA DE TEMAS ESPEC

IFICOS
Captulo 5
Problemas de Tarea
En esta seccion se presentan los problemas que los Olmpicos del a no pasado tuvieron
de tarea para el verano. Resolver estos problemas y escribirlos te puede ayudar a agarrar
practica en la b usqueda de ideas y en como las puedes expresar por escrito.

Algebra
1. Sea a
0
, a
1
, a
2
, ..., una sucesion denida recursivamente como a
0
= 0 y a
n+1
= 2a
n
+
2
n
. Prueba que n es potencia de dos si y solo si a
n
es potencia de dos.
2. En la expresion A =

2010 +
_
2010 +
_
2010 +
_
2010 +. . . +

2010 el n umero
2010 aparece 2010 veces. Encuentra el valor de A|, donde x| denota la parte entera
de x.
3. Sean x e y n umeros reales tales que 6x, 3+y
2
, 11+x, 14 y
2
son positivos. Halla
el maximo de la funcion f(x, y) =
_
(6 x)(3 +y
2
) +
_
(11 +x)(14 y
2
).
4. Halla todas las funciones f denidas en los naturales que satisfacen las siguientes
tres condiciones:
Para todo n, f(n) es un n umero natural.
Se cumple f(n +m) = f(n)f(m) para todos m y n.
Existe k tal que f(f(k)) = [f(k)]
2
.
Combinatoria
1. Cuantos n umeros de diez dgitos hay tales que para escribirlos se utilizan unica-
mente los dgitos 2 y 3 y que ademas no tienen dos dgitos 3 adyacentes?
61
62 CAP

ITULO 5. PROBLEMAS DE TAREA


2. Se escribe un entero en cada casila de un tablero de n las y n + 1 columnas.
Demuestra que se pueden tachar algunas columnas (posiblemente ninguna) de modo
que en cada la la suma de los n umeros que quedaron sin tachar sea par.
3. Demuestra que hay tantas maneras de repartir n monedas en m pilas, como maneras
de repartir n m monedas en pilas con m o menos que m monedas en cada una.
4. Tres n umeros a, b y c se escriben en un pizarron. En un movimiento se elige un
n umero y se remplaza por la suma de los otros dos menos uno (por ejemplo, si
al principio se elige a, entonces se cambia por b + c 1). Esta operacion se repite
varias veces hasta que se obtienen los n umeros 17, 1967, 1983. Los n umeros iniciales
pudieron haber sido 3, 3, 3? Pudieron ser 2, 2, 2?
5. Se tienen chas de domino, cada una de ellas con un 1 en un lado y un 1 en el
otro. Para que n es posible cubrir con estas chas un tablero de 5 n de modo que
el producto de los n umeros en cada columna y en cada la sea positivo?
Geometra
1. Sea ABC un triangulo y D el punto medio de BC, sea E el punto medio de AD y
F la interseccion de BE con AC. Encuentra
AF
FC
.
2. Sea ABC un triangulo rectangulo en B. Se tiene un punto P en el interior a distancia
12 de la hipotenusa AC, por el cual se van a trazar los siguientes segmentos paralelos
a los tres lados del triangulo ABC. El segmento paralelo a AC corta a AB en E y
a BC en F. El segmento paralelo a BC corta a AC en G. Finalmente, el segmento
paralelo a AB corta a AC en D. Si AB = 45 y BC = 60, cual es el area del trapecio
EDFG?
3. ABCD es un cudrilatero cclico con AB = AD. Las diagonales se intersectan en E,
F es un punto en AC tal que BFC = BAD. Si BAD = 2DFC, determina
BE
DE
.
4. Sea M un punto en el interior el triangulo equilatero ABC. Demuestra que MA
2
+
MB
2
= MC
2
si y solo si BMA = 150

.
5. Sea P un punto en el interior del triangulo equilatero ABC y sean D, E y F los pies
de las perpendiculares trazadas por P a los lados AB, AC y BC respectivamente.
Determina el lugar geometrico de los puntos P para los cuales existe un triangulo
con lados PD, PE y PF.
6. En el triangulo ABC, ABC = 60

. O es el circuncentro y H es el ortocentro. D es
un punto en BC tales que BD = BH. E es un punto en AB tal que BE = BO. Si
BO = 1, cual es el area del triangulo BDE?
7. Sea ABC un triangulo rectangulo en A. Sea X el pie de la altura correspondiente
a A y sea Y el punto medio de XC. Sobre la prolongacion del lado AB se toma el
punto D tal que AB = BD. Demuestre que DX es perpendicular a AY .
63
Teora de N umeros
1. Encuentra el menor entero n tal que el conjunto n, n +1, . . . , n +42 solo contiene
un cuadrado perfecto.
2. Encuentra todas las parejas de enteros positivos a y b tales que (a, b)
2
+ [a, b]
2
=
a
2
+ b
2
, donde (a, b) y [a, b] denotan el maximo com un divisor y el mnimo com un
m ultiplo de a y b respectivamente.
3. Sea n un n umero natural tal que n
2
n + 11 es el producto de cuatro primos no
necesariamente distintos. Encuentra el mnimo valor de n.
4. Sean n y p dos n umeros naturales con p primo y tales que pn + 1 es un cuadrado
perfecto. Demuestra que n+1 es la suma de p cuadrados perfectos, no necesariamente
distintos.
5. Determina la suma de los enteros positivos n para los cuales existe una k con:
1 +n +
n(n 1)
2
+
n(n 1)(n 2)
6
= 2
k
64 CAP

ITULO 5. PROBLEMAS DE TAREA


Captulo 6
Herramientas

Utiles

Algebra
Identidades

1
n(n+1)
=
1
n

1
n+1
1 + 2 + 3 + +n =
n(n+1)
2
1 + 2
2
+ 3
2
+ +n
2
=
n(n+1)(2n+1)
6
1 + 3 + 5 + + (2n 1) = n
2
1 +a +a
2
+ +a
n
=
a
n+1
1
a1
si a ,= 1
El truco de las razones Si
a
b
=
c
d
entonces
a+c
b+d
=
ac
bd
=
a
b
, es decir, podemos
sumar o restar arriba y abajo y obtenemos la misma fraccion.
Factorizaciones y productos notables
a
2
b
2
= (a +b)(a b)
a
2
+ 2ab +b
2
= (a +b)
2
a
2
2ab +b
2
= (a b)
2
= (b a)
2
a
n
b
n
= (a b)(a
n1
+an 2b + +ab
n2
+b
n1
)
Si n es impar, en la identidad anterior podemos substituir b por b y obtener
a
n
+b
n
= (a +b)(a
n1
a
n2
b + ab
n2
+b
n1
).
Factorizacion de Sophie-Germain: a
4
+ 4b
4
= (a
2
+ 2b
2
2ab)(a
2
+ 2b
2
+ 2ab)
x
3
+y
3
+z
3
3xyz = (x +y +z)(x
2
+y
2
+z
2
xy yz zx)
x
3
+y
3
+z
3
3xyz == (x +y +z)
(xy)
2
(yz)
2
(zx)
2
2
(x a)(x b) = x
2
(a +b)x +ab
(x a)(x b)(x c) = x
3
(a +b +c)x
2
+ (ab +bc +ca)x abc
65
66 CAP

ITULO 6. HERRAMIENTAS

UTILES
Las ultimas dos identidades nos permiten poner los coecientes de un polinomio de
grado dos o tres en terminos de sus races. Estas son algunas de las identidades de
Vietta.
Si tienes una ecuacion cuadratica ax
2
+bx+c, entonces sus races (es decir, los valores
de x para los cuales esta expresion se hace cero) son
b+

b
2
4ac
2a
y
b

b
2
4ac
2a
.
Exponentes y logaritmos
Propiedades de los exponentes:
a
0
= 1
a
b+c
= a
b
a
c
(a
b
)
c
= a
bc
(ab)
c
= a
c
b
c
a
1
=
1
a
El logartimo base a de un n umero b, que denotaremos por log
a
b es el n umero al cual
se tiene que elevar a para obtener b. Por ejemplo, log
2
8 = 3 pues al elevar 2 a la
potencia 3 obtenemos 8. A partir de las propiedades de los exponentes se obtienen
las siguentes propiedades para los logaritmos:
log
a
1 = 0
log
a
(b c) = log
a
b + log
a
c
log
a
(b
c
) = c log
a
b

log
a
c
log
b
c
= log
b
a
log
a
(
1
b
) = log
a
b
Desigualdades
Los n umeros reales se dividen positivos, negativos y el cero. Los positivos se colocan a
la derecha del 0 y los negativos a su izquierda. Una multiplicacion por 1 la podemos
pensar como una rotacion de 180

centrada en 0 de la recta real. Hacer dos veces


esto deja ja la recta real, por lo que (1)(1) = 1 y en general (a)(a) = a
2
.
Estas observaciones nos permiten armar que para cualquier real r tenemos r
2
0.
Las desigualdades se pueden sumar, si a > c y b > d entonces a +b > c +d. Multi-
plicando una desigualdad por un n umero positivo se preserva y por uno negativo se
invierte, es decir, si a > b entonces ra > rb si r > 0 y ra < rb si r < 0.
El valor absoluto de un n umero [x[ es su distancia al cero. Como las distancias son
positivas, si el n umero es negativo entonces se le debe cambiar de signo para hacerlo
positivo. Por ejemplo, [3[ = 3, [ 2,5[ = 2,5, [0[ = 0. Podemos calcular la distancia
entre dos n umeros por [x y[, lo cual es lo mismo que al mayor restarle el menor.
La desigualdad del triangulo dice que [a +b[ [a[ + [b[.
Algunas desigualdades algebr aicas:
67
r
2
0
Desigualdad entre la media geometrica y la media aritmetica. Para a, b 0,
tenemos que
a+b
2

ab. La igualdad se da solo cuando a = b.


Desigualdad del reacomodo. Si a
1
a
2
a
n
y b
1
b
2
b
n
entonces:
a
1
b
1
+a
2
b
2
+ +a
n
b
n
a
1
b
n
+a
2
b
n1
+ +a
n
b
1
Algunas desigualdades geometricas:
Dado un punto jo P, el conjunto de puntos cuya distancia a P es constante es
una circunferencia con centro en P. Los puntos interiores tienen una distancia
menor a P y los exteriores una mayor.
Dada una recta l y un punto P, la distancia mas corta de P a un punto de l se
hace cuando en l se elige el pie de la perpendicular desde P. A esta longitud se
le llama la distancia de P a l
Los lados de un triangulo son positivos. El lado mas grande es opuesto al angulo
mas grande, y el mas chico es opuesto al angulo mas chico.
En un triangulo con lados a, b y c se tienen las desigualdades del triangulo:
a +b > c, b +c > a, c +a > b. Este resultado es invertible, es decir, si los tres
n umeros reales positivos a, b y c cumplen que a +b > c, b +c > a y c +a > b,
entonces se puede hacer un triangulo de lados a, b, c.
Desigualdad de Ptolomeo. Para un cuadrilatero ABCD se tiene AC BD
AB CD+BC DA. La igualdad se da si y solo si los vertices del cuadrilatero
estan en una circunferencia (decimos que el cuadrlatero es cclico).
Notacion de suma
El smbolo

n
i=1
a
i
se utiliza para indicar la suma de los n umeros a
1
, a
2
, . . . , a
n
. Por
ejemplo, para escribir la suma de los primeros n n umeros podemos escribir

n
i=1
i y
para escribir la suma de los primeros n cuadrados podemos escribir

n
i=1
i
2
. El uso
de i es arbitrario, en realidad se puede usar cualquier smbolo que pensamos como
un contador. Algunos ejemplos:


10
i=1
i = 1 + 2 + 3 + 4 + 5 + 6 + 7 + 8 + 9 + 10 = 55


4
j=0
2j 1 = (1) + (1) + (3) + (5) + (7) = 15


8
k=5
k(10 k) = (5 5) + (6 4) + (7 3) + (8 2)
La notacion de suma tiene las siguientes propiedades:


n
i=1
1 = n


n
i=1
c a
i
= c

n
i=1
a
i


n
i=1
a
i
+b
i
=

n
i=1
a
i
+

n
i=1
b
i
68 CAP

ITULO 6. HERRAMIENTAS

UTILES
Parte entera Para un n umero real x, denotamos por x| al mayor entero que sea
menor o igual a x. Los siguientes ejemplos son ilustrativos: 10,2| = 10, 12| =
12,
_
25
6
_
= 4, 11,5| = 12,
_
79
_
= 8, 10000| = 31415 y 10000| =
31416.
La notacion x| se lee parte entera de x. La parte entera tiene las siguientes
propiedades:
Si x es un n umero entero, entonces x| = x.
Para cualquier real se tiene que x| x x + 1|.
La operacion parte entera es idempotente: x|| = x|.
Desigualdad del triangulo: x +y| x| + y|.
69
Combinatoria
Principios basicos de conteo Si existen m formas de hacer una cosa A y n formas
de hacer una cosa B entonces:
Hacer A o hacer B (pero solo una de las dos) se puede hacer de m+n formas.
Hacer A y luego hacer B se puede hacer de mn formas.
Permutaciones Si se tienen n objetos distinguibles, entonces se pueden poner en
la de n! formas distintas. La notacion n! se pronuncia n factorial y denota al
producto de los primeros n enteros. Ademas, denimos 0! = 1.
Combinaciones Si tenemos n objetos distinguibles, entonces podemos elegir k (0
k n) de ellos de
_
n
k
_
=
n!
k!(nk)!
formas. La notacion
_
n
k
_
se pronuncia n en k. A
los n umeros
_
n
k
_
se les conoce como coecientes binomiales.
Propiedades basicas de los coecientes binomiales
Denimos
_
n
k
_
= 0 si n es negativo, k es negativo o k > n.

_
n
k
_
=
_
n
nk
_
.
Formula de Pascal:
_
n+1
k+1
_
=
_
n
k
_
+
_
n
k+1
_
.

n+1
k+1
_
n
k
_
=
_
n+1
k+1
_
.

_
n
0
_
+
_
n
1
_
+ +
_
n
n
_
= 2
n
.
Las siguientes observaciones son sencillas, pero son muy poderosas. Muchas veces
estas observaciones nos ayudan a justicar con formalidad por que podemos o no
podemos hacer ciertas cosas.
Principio de las Casillas Esta es una de las armaciones mas intuitivas de las
matematicas. Tomando en cuenta su sencillez es una herramienta muy poderosa. El
Principio de las Casillas dice lo siguiente:
Si tenemos n + 1 objetos y n casillas para acomodarlos, entonces en alguna de las
casillas deben quedar al menos dos objetos.
El Principio de las Casillas admite las siguientes generalizaciones:
Si tenemos kn +1 objetos y n cajas, entonces al menos una de esas cajas tiene
al menos k + 1 objetos.
Si tenemos una innidad de objetos y los acomodamos en una cantidad nita
de cajas, entonces una caja tiene una innidad de objetos.
(Dual del principio de las casillas) Si tenemos n objetos y n+1 cajas, entonces
al colocar los objetos en las cajas una caja queda vaca.
70 CAP

ITULO 6. HERRAMIENTAS

UTILES
Ejemplo De tres personas, dos son del mismo sexo. Si tenemos 13 personas, hay dos
que nacieron el mismo mes.
Principio del Doble Conteo Si contando los objetos de un conjunto de una forma
obtenemos que son a y cont andolos de otra forma obtenemos que son b entonces
a = b.
Ejemplo Si tenemos n + 1 puntos alineados, estos hacen 1 segmento de longitud n,
2 de longitud n 1, 3 de longitud n 2, . . . , n de longitud 1. De modo que en total
hacen 1 + 2 + +n segmentos. Otra forma de contar cuantos segmentos hacen es
ver de cuantas formas podemos escoger dos de ellos. Esto se puede de
_
n+1
2
_
=
n(n+1)
2
formas distintas. Esto demuestra que 1 + 2 + +n =
n(n+1)
2
.
Principio de Invarianza Si tenemos un proceso que deja una variable ja (o hace
que tome siempre valores en un mismo conjunto), entonces con el proceso no podemos
llegar a una situacion en la cual esa variable sea distinta (o tome un valor fuera de
ese conjunto).
Ejemplo Comenzamos con el n umero 1. El proceso consiste en cada paso en sumar o
restar 2. Lo que se mantiene invariante es la paridad del n umero (siempre es impar),
de modo que con este proceso no podemos llegar a 2010.
Principio del Buen Orden Si tenemos un conjunto no vaco de n umeros enteros
positivos, o un conjunto nito de n umeros cualesquiera, entonces ese conjunto tiene
un mnimo, es decir, hay un n umero en el conjunto que es el mas chico de todos los
n umeros en el conjunto.
71
Geometra
Lneas y puntos
Asumimos como conocido el concepto de lnea y punto. Dados dos puntos A y B en
una lnea llamamos un rayo desde A a los puntos que estan en la lnea en un mismo
lado del punto A. A los puntos entre A y B le llamamos el segmento AB.
Dos lneas pueden o no intersectarse. Si no se intersectan, las llamamos paralelas.
Si se intersectan en un angulo de 90

entonces las llamamos perpendiculares. Si l es


perpendicular a m y m es perpendicular a n entonces l es paralela a n.
De hecho mas en general, si una lnea transversal corta a dos rectas y forma angulos
iguales, entonces estas dos rectas son paralelas. El regreso tambien es cierto, si dos
rectas son paralelas entonces los angulos que hacen con cualquier transversal son
iguales.
Tres puntos son colineales (o estan alineados) si existe una recta que pasa por los tres.
Tres rectas son concurrentes (o se intersectan) si existe un punto que se encuentre
en las tres.
Triangulos
Un triangulo es una gura con tres lados. La suma de los angulos internos de un
triangulo es 180

. En la siguiente gura el angulo

mide +. A este angulo se le


llama el angulo externo (o exterior) en A del triangulo ABC.
Decimos que un triangulo es isosceles si tiene al menos dos lados iguales. Decimos que
es equilatero si tiene sus tres lados iguales (y por lo tanto, un triangulo equilatero
es isosceles). En caso de que no suceda ninguna de estas opciones diremos que el
triangulo es escaleno.
72 CAP

ITULO 6. HERRAMIENTAS

UTILES
Otra forma de clasicar triangulos es por sus angulos. Un triangulo con todos sus
angulos menores a 90

se llama acutangulo. Un triangulo con un angulo de 90

se le
llama rectangulo. Si tiene un angulo obtuso se le llama obtusangulo.
En un triangulo rectangulo se conoce como hipotenusa al lado opuesto al angulo de
90

. A los otros dos lados se les conoce como catetos. El Teorema de Pitagoras nos
dice que la suma del cuadrado de los catetos es igual al cuadrado de la hipotenusa.
Es decir, en la gura siguiente a
2
+b
2
= c
2
.
Dos triangulos son congruentes si sus tres lados y sus tres angulos son iguales respec-
tivamente. Para vericar que dos triangulos son congruentes basta vericar alguna
de las siguientes condiciones:
Criterio LLL: Sus tres lados son iguales.
Crietrio ALA: Dos de sus angulos y el lado entre ellos son iguales.
Criterio LAL: Dos de sus lados y el angulo entre ellos son iguales.
Enfatizamos que la importancia de los criterios de congruencia es que con que se
cumpla alguna de las condiciones arriba mencionadas se tiene que el resto de los
lados y angulos son iguales.
Decimos que dos triangulos son semejantes si tienen sus tres angulos iguales y sus
lados guardan la misma proporcion. A esta proporcion la llamamos la razon de
semejanza entre los triangulos. Para vericar que dos triangulos son semejantes basta
vericar alguna de las siguientes condiciones:
Criterio LLL: Sus tres lados correspondientes guardan la misma proporcion
Crietrio AA: Dos de sus angulos son iguales.
Criterio LAL: Dos de sus lados guardan la misma proporcion y el angulo entre
esos lados es igual.
73
Si dos triangulos son semejantes en razon q, entonces la distancia entre parejas de
puntos correspondientes tambien estan en razon q, por lo que los permetros, alturas,
medianas, etc., tambien estan en razon q. Sin embargo, la razon entre las areas es
q
2
.
En un triangulo ABC se toman puntos D sobre AB y E sobre AC. El Teorema de
Tales dice que BC es paralela a DE si y solo si
AD
AB
=
AE
AC
.
A continuacion se muestran algunos triangulos importantes:
En un triangulo ABC se tienen las siguientes rectas importantes:
Mediana: Una mediana es la recta que va de un vertice al punto medio del lado
opuesto.
Bisectriz: Una bisectriz es la recta que pasa por un vertice y bisecta al angulo
del triangulo que esta en ese vertice.
Bisectriz Exterior: Una bisectriz exterior es la recta que pasa por un vertice y
bisecta al angulo exterior del triangulo que esta en ese vertice
Mediatriz: Una mediatriz es una recta perpendicular a un lado del triangulo
que pasa por el punto medio de ese lado.
Altura: Una altura es una recta perpendicular a un lado del triangulo que pasa
por el vertice opuesto.
Las medianas de un triangulo se cortan dos a dos en proporcion 1 : 2 y las tres
se intersectan en un punto, llamado el gravicentro del triangulo, que usualmente se
denota por G. Al triangulo formado por los tres puntos medios se le conoce como el
triangulo medial y es semejante al triangulo original en razon
1
2
, de modo que tiene
area
1
4
del triangulo original.
74 CAP

ITULO 6. HERRAMIENTAS

UTILES
Las tres bisectrices de un triangulo se intersectan en un punto llamado el incentro del
triangulo, que usualmente se denota por I. Existe una circunferencia que es tangente
internamente a cada uno de los lados del triangulo. A esta circunferencia la llamamos
el incrculo de ABC. Su centro es I, de modo que I equidista de los tres lados. Al
radio del incrculo lo llamamos el inradio del triangulo y usualmente se denota por
r.
La bisectriz interna y la externa por un mismo vertice son perpendiculares. Las dos
bisectrices externas por dos vertices y la bisectriz interna por el tercer vertice son
rectas concurrentes.
Las tres mediatrices de un triangulo se intersectan en un punto llamado el circun-
centro del triangulo, que usualmente se denota por O. Existe una circunferencia que
pasa por los tres vertices del triangulo. A esta circunferencia la llamamos el cir-
cuncrculo de ABC. Su centro es O, de modo que O equidista de los tres vertices.
Al radio del circuncrculo lo llamamos el circunradio del triangulo y usualmente se
denota por R.
Las tres alturas se intersectan en un punto llamado el ortocentro del triangulo, que
usualmente se denota por H. Al triangulo formado por los tres pies de las alturas se
le conoce como el triangulo ortico.
La mediatriz y la altura correspondientes a un mismo lado son paralelas.
75
Si en un triangulo dos de las siguientes rectas por un mismo lado (o vertice corres-
pondiente) son la misma: altura, mediana, mediatriz, bisectriz; entonces el triangulo
es isosceles con los otros dos lados iguales, y por tanto todas esas rectas (por ese
lado) son la misma. Por ejemplo, si la mediana por A tambien es perpendicular a
BC (i.e. es altura), entonces AB = AC y entonces la mediana tambien es la bisectriz
y mediatriz.
A partir de la observacion anterior, si dos de los puntos O, H, G o I coinciden
entonces el triangulo es equilatero.
El Teorema de la Recta de Euler nos dice que G, O y H estan alineados en cualquier
triangulo.
Circunferencias
Una circunferencia es el conjunto de puntos que equidistan de un punto jo P. La
distancia que tiene P a cada uno de esto puntos se conoce como el radio de la
circunferencia. Dados dos puntos A y B sobre la circunferencia, el segmento AB se
conoce como una cuerda. Si la cuerda pasa por el centro entonces se llama diametro
y mide dos veces el radio.
En una circunferencia, la medida del angulo que se hace con un vertice sobre la
circunferencia es la mitad del angulo central. Esto nos dice que angulos que abran
un mismo arco del mismo lado miden lo mismo. En particular, si un angulo abre un
diametro entonces el angulo es recto.
76 CAP

ITULO 6. HERRAMIENTAS

UTILES
Una tangente es una recta que intersecta a la circunferencia en un solo punto. Si T
es el punto de tangencia y RT es una cuerda por T, el angulo que hacen (llamado
semi-inscrito) es el mismo que el que abre a la cuerda RT en la circunferencia.
En particular, las tangentes son perpendiculares al radio que pasa por el punto de
tangencia. Las dos tangentes que se pueden trazar desde un mismo punto a una
misma circunferencia miden lo mismo.
Cuando dos circunferencias se intersectan, la recta que pasa por los dos puntos de
interseccion es perpendicular a la recta que une sus centros.
Cuadrilateros cclicos
Tres puntos no alineados determinan una unica circunferencia, de modo que la si-
tuacion en la que cuatro puntos A, B, C, D estan en una misma circunferencia es
especial. Cuando sucede esto, decimos que los cuatro puntos son concclicos o bien
que el cuadrilatero ABCD es cclico.
Supongamos ademas que AC se intersecta con BD en el interior del cuadrilatero
en un punto P y que AB se intersecta con CD en un punto S. Entonces todas las
siguientes armaciones son equivalentes, es decir, tan solo con vericar que sucede
una de ellas, inmediatamente sabemos que se cumplen las demas:
El cuadrilatero ABCD es cclico.
Existe un punto O que equidiste a A, B, C y D.
A+C = 180

B +D = 180

ACB = ADB
BAC = BDC
77
CBD = CAD
DCA = DBA
Potencia de un punto interior: PA PC = PB PD
Potencia de un punto exterior: SA SB = SC SD
Igualdad de Ptolomeo: AC BD = AB CD +BC DA
Usamos una terminologa similar cuando tenemos mas puntos sobre una misma cir-
cunferencia, por ejemplo, podemos hablar de hexagonos cclicos o decir que tenemos
n puntos concclicos.
Trigonometra
En un triangulo ABC con a = BC, b = CA y c = AB con circunradio R se cumple
lo siguiente:
Ley de senos:
a
sen A
=
b
sen B
=
c
sen C
= 2R
Ley de cosenos: c
2
= a
2
+b
2
2ab cos C
Ademas, tenemos las siguientes identidades trigonometricas:
sen 0

= cos 90

= 0
cos 0

= sen 90

= 1
sen(x) = sen x
cos(x) = cos x
sen x = cos(90

x)
Seno de una suma: sen(x +y) = sen xcos y + cos xsen y
Coseno de una suma: cos(x +y) = cos xcos y sen xsen y
Tangente de una suma: tan(x +y) =
tan x+tan y
1tan xtan y
Observa que a partir de estas identidades puedes encontrar muchas otras formulas
mas, por ejemplo, si en la formula de seno de la suma de angulo pones el mismo
angulo obtienes sen(2x) = 2 sen xcos x. O si quieres encontrar el coseno de una resta
x y entonces puedes usar la de coseno de la suma y cambiar y por y y usar
las otras identidades para obtener cos(x y) = cos xcos(y) sen xsen(y) =
cos xcos y + sen xsen y.
78 CAP

ITULO 6. HERRAMIENTAS

UTILES
Teora de N umeros
Clasicacion de los n umeros
Llamamos enteros a los n umeros con los cuales podemos contar cosas y sus negativos,
por ejemplo 1, 2, 3, 10, etc. Llamamos n umeros racionales a los n umeros que se
pueden escribir como una division de enteros. A los n umeros reales los podemos
pensar como todos los n umeros que se pueden escribir en expansion decimal. A los
reales que no son racionales, como , los llamamos irracionales. Los smbolos para
denotar a los conjuntos de n umeros naturales, enteros, racionales y reales son N, Z,
Q y R respectivamente. El conjunto de n umeros irracionales no tienen su smbolo,
pero los podemos denotar por R Q (los reales que no son racionales).
Ocasionalmente a cualquiera de estos conjuntos los podemos limitar un poco mas
especicando que se habla de positivos, negativos, no negativos, etc.
Divisibilidad
Para dos n umeros enteros a y b diremos que a divide a b si existe un entero k para
el cual b = ka. Tambien esto se puede decir como b es m ultiplo de a, a es un divisor
de b, entre otras. La notacion para a divide a b es a[b. A partir de este momento solo
nos referiremos a n umeros enteros, salvo que explcitamente se diga lo contrario.
Por denicion, 0 es divisible entre cualquier n umero, pues la k correspondiente es 0.
As mismo, 1 divide a todo n umero. A los n umeros divisibles ente 2 los llamamos
pares y a lo n umeros enteros que no son pares los llamamos impares (o menos
frecuentemente, nones). Como 0 es divisible entre 2, entonces 0 es par.
A los n umeros que tienen exactamente dos divisores positivos los llamamos primos.
De esta forma, 2 es primo, 13 es primo, pero ni 6 ni 1 lo son.
La divisibilidad cumple las siguientes propiedades para cualesquiera n umeros enteros
a ,= 0, b, c, k, l:
a[a
Si a[b y b[c entonces a[c.
Si a[b y b[a y a y b son positivos entonces a = b.
Si a[b y a[c entonces a[bk +cl.
Si p es primo y p[n
k
entonces p[n.
Para dos n umeros denimos su mnimo com un m ultiplo como el menor entero po-
sitivo que es m ultiplo de ambos. De manera similar, denimos su maximo com un
divisor como el mayor entero positivo que los divide. Denotamos a estos n umeros por
[m, n] y (m, n) respectivamente. Si dos n umeros tienen como maximo com un divisor
a 1 entonces los llamamos primos relativos.
Si k es el maximo com un divisor de m y n entonces k se puede expresar como
combinacion lineal de m y n, es decir, existen enteros a y b para los cuales k =
am+bn.
79
En muchas ocasiones un n umero a no divide a un n umero b, sin embargo, podemos
expresar a b de la forma b = qa + r con 0 r < a , es decir, podemos poner a b
como un m ultiplo de a mas un residuo peque no r que esta entre 0 y a 1.
Algunos criterios de divisibilidad
Todo n umero es divisible entre 1.
Un n umero es divisible entre 2 si y solo si su ultimo dgito es par.
Un n umero es divisible entre 3 si y solo si la suma de sus dgitos es m ultiplo de
3.
Un n umero es divisible entre 4 si y solo si el n umero formado por sus ultimos
dos dgitos es m ultiplo de 4.
Un n umero es divisible entre 5 si y solo si su ultimo dgito es 0 o 5.
Un n umero es divisible entre 6 si y solo si es divisible entre 2 y 3.
Un n umero es divisible entre 8 si y solo si el n umero formado por sus ultimos
tres dgitos es m ultiplo de 8.
Un n umero es divisible entre 9 si y solo si la suma de sus dgitos es m ultiplo de
9.
Un n umero es divisible entre 11 si al sumar y restar alternadamente sus dgitos
se obtiene un n umero divisible entre 11.
Un buen ejercicio para practicar los criterios de divisibilidad es ver entre cuales
n umeros que tengan criterio es divisible el n umero 12345678.
La cantidad de m ultiplos de k que existen desde 1 hasta n es
_
n
k
_
.
Teorema Fundamental de la Aritmetica
El Teorema Fundamental de la Aritmerica dice que todo entero se puede descom-
poner como producto de primos de manera unica salvo el orden de los factores. En
smbolos, si n es un natural entonces existen k primos p
1
, p
2
, . . . , p
k
y exponentes
a
1
, a
2
, . . . , a
k
para los cuales n = p
a
1
1
p
a
2
2
p
a
k
k
.
Descomponiendo a m en producto de primos como m = p
a
1
1
p
a
2
2
p
a
k
k
se tienen
las siguientes formulas:
N umero de divisores de m: (a
1
+ 1)(a
2
+ 1) . . . (a
k
+ 1)
Suma de los divisores de m:
p
a
1
+1
1
1
p
1
1
p
a
2
+1
2
1
p
2
1
. . .
p
a
k
+1
k
1
p
k
1
Primos relativos con m que estan entre 1 y m: m(1
1
p
1
) . . . (1
1
p
k
)
Ademas, si tambien descomponemos a n en producto de primos como n = p
b
1
1
p
b
2
2

p
b
k
k
, pensando en que algunos de los exponentes pueden ser 0 entonces tenemos:
Maximo com un divisor de m y n: p
mn(a
1
,b
1
)
1
p
mn(a
2
,b
2
)
2
p
mn(a
k
,b
k
)
k
80 CAP

ITULO 6. HERRAMIENTAS

UTILES
Mnimo com un m ultiplo de m y n: p
max(a
1
,b
1
)
1
p
max(a
2
,b
2
)
2
p
max(a
k
,b
k
)
k
Congruencias
Diremos que a y b son congruentes modulo un n umero n si el residuo de dividir
ambos n umeros entre n es el mismo. Esto es equivalente a que n divida a a b.
La notacion para esto es a b mod n. Por ejemplo, 2009 2011 mod 2, 10 3
mod 7, 100 1 mod 9, 4 1 mod 5.
Ver a los n umeros as nos permite enfocarnos unicamente en una cantidad nita de
terminos, pues los posibles residuos de dividir un n umero entre n son solamente 0,
1, 2, . . . , n 1.
Dado un modulo jo, las congruencias se pueden tratar como igualdades en la ma-
yora de los casos. Se pueden sumar, restar y multiplicar, pero no siempre se pueden
dividir. Esto y otras propiedades basicas se enlistan a continuacion:
a a mod n
Si a b mod n entonces b a mod n
Si a b mod n y b c mod n entonces a c mod n
Si a b mod n y c d mod n entonces
a +c b +d mod n
a c b d mod n
ac bd mod n, en particular, a
k
b
k
mod n
Diremos que a tiene inverso modulo n si existe un n umero b para el cual ab 1
mod n. Si a y n son primos relativos entonces a tiene inverso modulo n. En particular,
dado un primo p, todos los n umeros que no son m ultiplos de p tienen inverso modulo
p.
Si a tiene un inverso b modulo n y ac d mod n entonces c bd mod n. Esto es
mas o menos dividir entre a modulo n.
Las congruencias sirven para encontrar facilmente el residuo que deja un n umero al
dividirlo entre otro. Para que una igualdad entre enteros se cumpla, se debe cumplir
en cualquier modulo. Esto quiere decir que si no existen soluciones para una ecuacion
en enteros para una congruencia, entonces no existe la solucion en los enteros.
Finalmente, se enlistan algunos teoremas mas avanzados de las congruencias:
Peque no Teorema de Fermat: Si p es primo y a no es m ultiplo de p entonces
a
p1
1 mod p.
Teorema de Wilson: Si p es primo, entonces (p 1)! 1 mod p.
Teorema de Euler: Si (n) denota la cantidad de primos relativos con n que
estan entre 1 y n, y a es primo relativo con n entonces a
(n)
1 mod n.

Vous aimerez peut-être aussi